LEAP Practice Exam

Ace your homework & exams now with Quizwiz!

A social worker is trying to complete all her paper work before she leaves for vacation the following day. It is midnight, and she is aware that she needs to get home, pack her clothes and get some sleep. She decides that because she only has two more notes to make in a client's file, she will take the file home and make the entries first thing in the morning. In this situation, the social worker is A. Risking an unethical situation B. Managing her time properly C. Doing what is common practice D. Violating the law

A- Risking an unethical sitatuion. The social worker may or may. Not be doing what is common practice (answer C), but she is definitely risking an unethical situation. She risk breaching confidentiality for taking the field to her home where others might see them. Managing her time properly (answer B) is not correct b/c she needs to manager her time in ways that will not put her or her client at risk. Violating the law (answer D) is not correct as she is not violating the law.

A family comes to see a social worker in a child welfare agency to discuss family problems. The father discusses how he was sexually abused by his father. The following is the most likely situation in this family. A Sexual abuse in this family B. Depression in the children D. Depression in the father D. A passive mother

A- Sexual abuse in this family. Men who have been sexually abused by a father are significantly more likely to sexually abuse their own offspring. The question makes not reference to depression (answers B and C) in any of the family members or the functioning of the mother (answer D), which makes those three answer incorrect.

During a family therapy session, the teenage daughter begins accusing her mother of being uncaring and neglecting her needs. The mother repeatedly responds very passively and does not defend herself, while the father continually attempts to take control of the session. The social worker needs to examine the family situation for A. Sexual abuse B. Individuation issues C. Triangulation D. Substance Abuse

A- Sexual abuse. In incestuous families, the father is often highly controlling and the mother is often passive. Individuation issues (answer B) is not a correct answer b/c there appear to be far more serious problem that just the daughter attempting to become independent by separating herself from the family. Triangulation (answer C) is not correct as the dynamics of the scenario do not suggest triangulation issues. Substance abuse (answer D) is not correct b/c there is no information in this scenario to suggest substance abuse.

In an initial interview, a client becomes belligerent and threatening for no apparent reason. The social worker should FIRST A. Share his or her concern with the he client B. Firmly tell the client to quite down C. Call security D. Ask the client to re-schedule at another time

A- Share his or her concerns with the client. The question asks for the FIRST thing the social worker should do. When the client exhibits a sudden change in behavior, the social worker's sharing his or her concern with the client is preferable to firmly telling the client to quiet down (answer B) or asking the client to leave (answer D) b/c the former response may increase the client's self- awareness. This is almost always preferable to a directive. Additional action may also be needed, such as calling security (answer C).

A client in a family agency requests to see her records. The social worker should A. Show the records if it will not harm the client B. Politely refuse C. Obtain a legal order permitting the social worker to show the records to the client D. Try to talk the client out of wanting to see the records

A- Show the records if it will not harm the client. Client's have the right to access information in their file if that information would not be harmful to them. Politely refusing access (answer B) is not correct b/c refusing to show the client her records would violate her rights. Obtaining a legal order permitting her to show the records to the client (answer C) is not correct b/c obtaining a legal order is not necessary. Attempting to talk the client out of wanting to see her records (answer D) is not correct b/c this action violates the client's right to self- determination.

A client complains to her social worker that she has constant headaches that are so painful that she often misses work. She has had a complex physical examination and all of her test results are in the normal range. She has tried various migraine medications with mixed success. She takes pain medication (both prescription and over the counter) but still complains of debilitating headaches. Which of the following diagnoses best describes the client? A. Somatic symptom Disorder B. Possible brain tumor C. Conversion Disorder D. Illness Anxiety Disorder

A- Somatic Symptoms Disorder. In the absence of other medical conditions, the client has a significant somatic symptom that is distressing and creating a significant disruption to the client's life. Possible brain tumor (answer B) is not correct b/c this likely would have been detected during her medical examination. Conversion Disorder (answer C) is not correct as there is no voluntary motor or sensory alterations. Illness Anxiety Disorder (answer D) is also not correct. The client has an actual somatic symptom and not just a preoccupation with having a serious illness.

While holding a first session for a group of newly divorced client's, a woman begins discussing her anger with men in general. One man in the group begins to appear uncomfortable, while another starts to argue with the woman. What is the MOST LIKELY reason for the men's response? A. Transference B. Interpersonal conflict C. Projection D. Complimentary Ego- states

A- Transference. It is likely that the men's responses to the woman represent a shifting of emotions they experienced with a significant woman from their past, like a mother, to the female member of the group who is expressing anger with men. Interpersonal conflict (answer B) is not correct b/c it is unlikely that the men have an interpersonal conflict with this woman since they just met her. Projection (answer C) is not correct since projection is a defense mechanism by which one's own negative characteristics are denied and instead seem as being characteristics of someone else. Complimentary Ego- states (answer D) is not correct, as complimentary ego- states refers to the men having the same ego involvement, which is why they respond similarly, and this is unlikely.

A young man was recently arrested for looking in the windows of neighbors as they dressed in the morning. Which of the following diagnoses is MOST LIKeLY? A. Voyeuristic Disorder B. Exhibitionistic Disorder C. Fetishistic Disorder D. Frotteuristic Disorder

A- Voyeuristic Disorder. Voyeuristic is a Paraphilic Disorder associated with sexually arousing urges or fantasies involving observing an unsuspecting individual who is naked undressing, or involved in sexual activity. Exhibitionisitic Disorder (answer B) is not correct b/c it is characterized by intense and recurring urges, fantasies, or behaviors that are sexually arousing (minimum 6 months) around exposing one's genitals to people the individual does not know. Fetishistic Disorder (answer C) is not correct b/c it is characterized by intense and recurring urges, fantasies, or behaviors (minimum of 6 months) that are sexually arousing and are focused on inanimate objects. Frotteristic Disorder (answer D) is not correct b/c it is characterized by intense and recurring urges, fantasies, or behaviors (minimum of 6 months) that are sexually arousing and focused on touching or rubbing against a person who has not given consent.

In the case of transference, a client is seeing the social worker as A. A past figure in his life B. A negative connotation C. A positive connotation D. A bad relationship

A- A past figure in his life. Transference involves the client's response to a person in the present as if he or she were someone from the past. This response can have a positive or negative connotation (answer B and C), or could have a bad relationship (answer D). When the social worker responds to the client in this manner, the term used is "countertransference".

In a recent incident at a local social service agency, a mentally ill client stabbed a social worker. The incident has produced extreme tension and fear in the other employees. The supervisor's FIRST step in easing tension would be to A. Hold a group meeting to discuss fears B. Place security throughout the building C. Reassure staff of safety measures D. Allow staff to take a mental health day

A- Hold a group meeting to discuss fears. The question asks for the FIRST step the supervisor should take. When a traumatic event such as this occurs, those affected need assistance in processing the event. Part of the group meeting may include reassuring staff of safety measures (answer C). After the employees have had the opportunity to discuss their fears decisions can be made, often with the assistance of the supervisor, relative to how to handle the situation att he present, such as taking a mental health day (answer D), or placing security throughout the building (answer B).

Ethnographic interviewing is one of the ways in which a social worker can ascertain the ____ of a client's cultural experiences A. Meaning B. Geographic origin C. Depth D. Political relevance

A- Meaning. This is a recall question.

A social worker, Ethan, is proving supervision to another social worker, Jay, who is working on an advanced license. Jay becomes sexually involved with a client and tells Ethan about the relationship. Ethan's responsibility is to A. Report Jay to the state Social Work licensing board B. Work the Jay to understand his behavior and to refer the client to another social worker C. See the client himself D. Insist Jay seek treatment

A- Report Jay to the state Social Work licensing board. Social workers are ethically bound by the NASW Code of Ethics to report such incidents to their licensing board. Working with Jay to understand his behavior and referring the client to another social worker (answer B) is not correct b/c Ethan is not Jay's social worker and it is not appropriate for him to intervene in this way as Jay needs to be reported. Seeing the client himself (answer C) is not correct as Ethan's seeing the client would be considered unethical. Insisting that Jay seek treatment (answer D) is not correct b/c it will be the licensing board that will determine the course of action for a social worker who is sexually involved with his client.

Substance intoxication differs from substance withdrawal in terms of A. The timing of symptoms B. The amount of substance in the system C. The individual's tolerance level D. They are synonymous

B- The amount of substance in the system. The symptoms of substance withdrawal are associated with a decrease in the amount of substance in the system from the intoxication stage. The timing of symptoms (answer A) and individual's tolerance levels (answer C) are not correct answers b/c these are not criteria which differentiate substance intoxication from substance withdrawal. They are synonymous (answer D) is incorrect as the two terms are not synonymous.

A young woman who is receiving counseling states that she is interested in obtaining her Ph.D in psychology. Her social worker is aware that her financial situation is such that a long- term educational plan does not seem currently possible. The FIRST part of the social worker's intervention plan at this point needs to be A. Helping the client to see the difficulties with the goal B. Working with the client to find resources to earn a Ph.D C. Suggesting alternatives to the Ph.D D. Referring the client to a career counselor

B- working with her to find resources to earn a Ph.D. This is the best answer to this question, as it is the responsibility of social workers to assist their client to access the resources they need to achieve their goals. The social worker may help her see the difficulties with the goal (answer A) and discuss alternatives to the Ph.D (answer D) as part of helping her find resources. If needed, the social worker can also refer her to a career counselor (answer D).

When evaluating a newborn infant, the most positive score on the APGAR is A. 6 B 8 C. 10 D. 15

C- 10. The APGAR test determines the immediate physical well- being of a newborn baby. The APGAR score only goes to 10, which is the best score. Physicians generally become concerned with initial scores of 7 and below.

A social worker in a large agency has several workers whom he supervises. All of the following are responsibilities of the social work supervisor EXCEPT A. Evaluating the work of the supervisee B. Making work assignments C. Providing counseling to the supervisee D. Educating the supervisee about agency policy and procedure

C- Providing counseling to the supervisee. The social work supervisor is responsible for all of the tasks listed except answer C. Supervisors are not to provide counseling services to supervisees b/c this constitutes a dual relationship, which is a violation of the NASW Code of Ethics. This question asks for which answer is NOT the responsibility of the social worker supervisor.

A young woman has been diagnosed with Borderline Personality Disorder. It can be expected that she will display A. Grandiose behavior B. Excessive scrupulousness C. Splitting D. Lack of intimate relationships

C- Splitting. Neither grandiose behavior (answer A) nor excessive scrupulousness (answer B) is characteristic of Borderline Personality Disorder. Although it is often true that individuals with this diagnosis have problems in their intimate relationships (answer D), this does not mean that they do not have intimate relationships.

All of the following are MAO Inhibitors EXCEPT A. Marplan B. Parnate C. Nardil D. Sinequan

D- Sinequan. Marplan (answer A), Parnate (answer B), and Nardil (answer C) are incorrect answers as they are medications used in the treatment of depression.

Which of the following is NOT recommended for Breathing Related Sleep Disorders? A. Nasal strips B. Weight loss C. Stimulants and tricyclics antidepressants D. Taking short naps during the day

D- Taking short naps during the day. However, taking naps is the recommended treatment for Narcolsepsy. All of the other answer options, nasal strips (answer A), weight loss (answer B), and stimulants and tricyclics antidepressants (answer C) are recommended for Breathing Related Sleep Disorders.

In a session with a marital couple, the husband announces that he is planning to proceed with a divorce. The wife becomes extremely upset and threatens to commit suicide. The husband's response is to say, "Go right ahead and do it." Then he walk out of the office. The social worker should FIRST A. Arrange for another couple session with the wife and husband B. Set up an individual appointment for the wife C. Call the humans and have him return D. Work with the wife to ensure her safety

D- Work with the he wife to ensure her safety. The social worker must determine that the wife is stable and safe before she leaves the session. When this has been accomplshed, the social worker can attend to scheduling another appointment for the wife (answer B). B/c the husband has clearly expressed his intention to divorce, it would be inappropriate for the social worker to either call the husband and have him return (answer C) or to arrange for another couple session with the wife and husband (answer A).

A 50- year- old man was found wandering in a city three hundred miles from his home. He was taken by the police to a hospital to be examined. The client could not remember his name and did not know how he got there. He remembered the name of the city in which he lived and the police were able to find relatives there. The relatives reported that he had not planned this trip and that he has been functional before this occurrence. What is MOST LIKElY occurring? A. Early- onset Alzheimer's B. Dementia C. Dissociative Identify Disorder D. Dissociative Amnesia

Dissociative Amnesia. This condition involves an inability to recall important personal information. "With dissociative fugue" is a specifier for Dissociative Amnesia that involves travel away from home that is sudden and unexpected, that is accompanied with a loss of memory and confusion about one's personal identify. Early- onset Alzheimer's (answer A) is not correct b/c this condition progresses slowly and is more debilitating than the symptoms described in this scenario. It is a type of dementia. Dementia (answer B) is not correct b/c Dementia is a deterioration of a mental processes, usually characterized by memory loss, personality change and impaired judgement and ability to think abstractly. Dissociative Identity Disorder (answer C) is not correct b/c the client has not developed two or more district personalities.

While undertaking grief counseling with a woman whose child has recently died, the social worker is most likely to do the following in the first session A. An assessment B. An intervention C. Testing D. Prescribe medication

A- An assessment. It is important that a social worker take the time in an initial session to carefully assess the client. This assessment will provide the basis for developing an effective treatment plan. An intervention (answer B) is not correct b/c the intervention will come after the assessment and development of a treatment plan. Testing (answer C) is not correct b/c testing is typically not standard in grief counseling, especially in the case of a child's death. If the client expresses symptoms of severe depression, the social worker may consider a depression inventory, but the decision would likely not occur in the first session. Prescribing medication (answer D) is not correct b/c grieving is a natural process and social workers do not prescribe medication.

An 18- year- old man has been referred to a mental health agency for services. He reports serving time in a detention center when he was 14 years old, a history of lying because it "makes him feel good" and "annoys others"; incidences of losing his temper, which have resulting in physical assaults; and deliberate destruction to others' property. According to the DSM-5, which of the following diagnoses BEST describes the client? A. Antisocial Personality Disorder B. Oppositional Defiant Disorder C. Conduct Disorder D. Unspecified Personality Disorder

A- Antisocial Personality Disorder. According the the DSM-5, deliberate destruction of others' property, lying, and physical assaults are all indicators for Antisocial Personality Disorder for an adult whose behaviors were manifested by age 15. Oppositional Defiant Disorder (answer B) and Conduct Disorder (answer C) are both childhood disorder, but since the client is now and adult, they are not the "best" diagnoses. The client's symptoms are specified and therefore answer "D" is incorrect.

When a social worker uses an empathic response, she is attempting to A. Build a relationship with the client B. Provide accurate listening C. Develop confidence D. Be sympathetic

A- Build a relationship with the client. Research indicates that social worker empathy is a critical factor in the quality of the therapeutic relationship. Providing accurate listening (answer B) is not the best answer as providing accurate listening is only a part of building a relationship with the client. Developing confidence (answer C) is not correct b/c developing confidence is a function of the client, not the social worker. Being sympathetic (answer D) is not correct b/c being sympathetic means feeling sorry for the client, which is not an empathic response.

Female genital pain during sexual intercourse that is recurring or persistent and is not the result of lubrication or vaginal spasms is most associated with which of the following? A. Genito- Pelvic/ Penetration Disorder B. Female Sexual Interest/ Arousal Disorder C. Female Orgasmic Disorder D. None of the above

A- Genito- Pelvic/ Penetration Disoder. This is a definitional question. Female Sexual Interest/ Arousal Disorder (answer B) Female Orgasmic Disorder (answer C) is not correct b/c the client's symptoms are not related to achieve orgasm.

In Task- Centered Therapy the social worker develops a contract in conjunction with the client through A. Goal- setting B. Problem- solving C. Mutual acceptance D. Multi- tasking

A- Goal- setting; the hallmark of Task- Centered Therapy. Problem solving (answer B), mutual acceptance (answer C), and mutli- tasking (answer D), are incorrect answers as they are not part of the goal- setting process in Task- centered Therapy.

A 5- year- old girls and a 4- year- old boy are found by the boy's mother in the garage with their pants down. They do not appear to be engaging in sexual activity. The mother calls her social worker for advice. The BEST way for the social worker to respond to the mother is to A. Normalize the situation, as this is a common age for this type of activity B. Suggest the boy have a psychiatric exam C. Have the parents of the children come in together D. See the boy individually

A- Normalize the situation, as this is a common age for this taupe of activity. This is a question dependent on knowledge of child development. The other three answers, suggesting the boy have a psychiatric exam (answer B), having the parents of the children come in together (answer C), and seeing the boy individually (answer D), are all not correct as the actions describe are not warranted based on the information presented in the question.

A social worker who is the administrator of a large organization that works to find adequate housing for homeless client's has been conducting research to determine the effectiveness of the agency. Which of the following is another term for summarize program evaluation? A. Outcome analysis B. Formative program evaluation C. Process analysis D. Cost effectiveness analysis

A- Outcome analysis. Formative program evaluation (answer B) is not correct b/c formative program evaluation evaluates a program from the planning stage through the implementation stage. Process analysis (answer C) is not correct, as process analysis is simply another term for formative program evaluation. Cost effectiveness analysis (answer D) is not correct cost effectiveness analysis refers to comparing the cost of a program to the program output (cost per unit).

An example of primary prevention is A. Premarital counseling B. Individual treatment C. Halfway house D. EKg

A- Premarital counseling. Primary prevention has reference to an intervention or program designed to prevent the development of a problem. The purpose of premarital counseling is to prevent the development of marital problems. Individual treatment (answer B) and halfway house (answer C) are incorrect answer b/c individual treatment and halfway houses are intervention used with individuals who are already manifesting problems. EKG (answer D) is not correct b/c an EKG is a means of assessing whether an individual already has a heart problem, but is not able to prevent the hart problem.

A major role of a group leader in a Therapeutic Group is to A. Support the natural process of the group B. Stay in the here- and- now C. Ensure that everyone has the opportunity to speak D. Alleviate conflict

A- support the natural process of the group. This is the primary role of the leader of the therapeutic group. Staying in the here- and- now (answer B) is not correct b/c the group leader would be controlling the group of he or she forced the group to stay in the here and now, as the natural process will likely cover past as well as current issues. Ensuring that everyone has the opportunity to speak (answer C) is not correct b/c the group leader does not have to ensure that everyone has the opportunity to speak. Group members may participate at their own comfort level. Alleviating conflict (answer D) is not the best answer b/c conflict may arise in the group, which can be healthy and teach the group members how to resolve conflict.

The primary factor for a social worker to consider when setting up a behavioral treatment program for a client is A. The nature of the dysfunctional behavior B. The outcome desired C. The dysfunctional thoughts behind the behavior D. The triggers for the behavior

A- the nature of the dysfunctional behavior. This is the starting point for any treatment program, behavioral or otherwise. The problem behavior must be clearly defined before an effective treatment program can be devised. The desired outcomes (answer B) is not the best answer b/c the desired outcomes should be identified after the baseline data has been taken. The dysfunctional thoughts behind the behavior (answer C) and the triggers for the behavior (answer D) are not correct b/c while the dysfunctional thoughts and the triggers will be important issues for the social worker to explore, an accurate description of the target behavior is the first step in setting up a behavioral program.

During a marital session,, a husband begins to describe his wife's habit of leaving the dishes in the sink. He says she ahs a knack for forgetfulness and hurrying to get things done. The worker's BEST response is to A. Say nothing B. Encourage the wife to express her feelings C. Interrupt the husband and allow the wife to talk D. Ask the husband about related patterns in his childhood home

B- Encourage the wife to express her feelings. When one member of a couple has shared his or her perspective about the other person, it is important that the latter be given the opportunity to express his or her feelings about what has been said. Saying nothing, (answer A), and interrupting the husband (answer C), are not correct answers to this question as they are not sound therapeutic skills. Asking the husband about related patterns in his childhood home (answer D) is not appropriate to the discussion at this time.

A social worker who is an administrator of an agency is evaluating his agency using a formative program evaluation. Which of the following statements describes a formative program evaluation? A. Eviction of the outcome B. Evaluation of the process C. Evaluation of effectiveness D. Validity of program.

B- Evaluation in process. The formative program evaluation evaluates the program from the planning stage through the implementation stage. Evaluation of the outcome (answer A) is not correct b/c the evaluation of the outcome is called summarize program evaluation. Evaluation of the effectiveness (answer C) is not correct as an evaluation of effectiveness involves hard data that proves that a program achieved the desired goal. Validity of program (answer D) is not correct b/c validity refers to the extent to which a procedure or evaluation is able to measure what it is intended to measure.

A social worker is employed in a family agency. His supervisor states that he would like to hear a recording of the social worker's next session with a particular family. In order to make this recording, the social worker must A. Inform the family of the recording B. Explain the purpose of the recording to the family and receive written permission C. Obtain verbal permission from the family D. Record the sessions since the supervisor is part of the agney

B- Explain the purpose of the recording to the family and receive written permission. This is an ethical, and often legal, requirement of social worker's prior to recording a session. Informing the family of the recording (answer A) is not correct b/c the family must give permission to be recorded, not be just informed. Obtaining verbal permission from the family (answer C) is not correct b/c verbal permission is not adequate. Recording the sessions since the supervisor is part of the agency (answer D) is not correct b/c written permission is required.

A school social worker is aware of several elementary school students whose parents are divorced. She is considering forming a group with these students. In what way would a group be beneficial? A. It would let the student hear other students' similar stories of experiences that have happened in their homes B. It would let them know that they are not alone. C. It would give the student ideas about how to get their parents back together. D. It would reduce the students' trauma

B- It would let them know that they are not alone. Often young children are not able to talk about their parents' divorce b/c of inability to find the words or embarrassment. Letting the students hear other students' similar stories of experiences that have happened in their homes (answer A) is not correct b/c the stories may re-traumatized the students. Giving the students ideas about how to get their parents back together (answer C) is not correct b/c, although almost every child of divorce would like his or her parents to get back together, it is usually futile and could give them false hope. Reducing the students' trauma (answer D) is not correct b/c one of the risks of this type of group is that it may actually re-traumatized a student.

A 66- year- old man makes an appointment to see a social worker in the senior citizen's center. He reports that he has been feeling very "blue" and that he lacks enjoyment in doing most things. He states that this has been going on for almost six months. In gathering his history, the social worker discovers that the client retired from a job of forty years approximately a year ago and has been traveling up until about seven months ago. It is likely that the client is A. Chemically depressed B. Just starting to deal with retirement C. Feeling useless D in an acute crisis state

B- Just staring to deal with retirement. The client's symptoms appeared shortly after his return from an extended trip he made following retirement. The timing suggests that the trip may have delayed his dealing with issues associated with his retirement. Chemically depressed (answer A) is not correct b/c in order to determine if the client is chemically depressed, a complete evaluation would have to be conducted. Feeling useless (answer C) is not the best answer b/c it is presumptuous. In an acute crisis state (answer D) is not correct b/c the client is not in an acute crisis state, as the feelings have been going on for six months.

A client expresses her aggravation with her mother-in-law b/c of her interference with the raising of the grandchildren. The client and her husband have rules that their children must follow and one of the rules is no ice cream. One of the children is lactose intolerant and suffered a severe reaction if he east milk products. The other children are not allowed to eat ice cream b/c it would be unfair. When the children's grandmother takes them out for the day she lets them all eat ice cream and asks them not to tell their parents. Which of the following defense mechanism is the grandmother using? A. Reaction Formulation B. Passive- aggression C. Compensation D. Rationalization

B- Passive aggression. The grandmother is expressing her disagreement and hostility toward the parents in an indirect and un assertive manner to avoid confrontation. Reaction Formulation (answer A) is not correct b/c. Reaction Formulation means behaving in a way that is opposite from one's unconscious beliefs or feelings. Compensation (answer C) is not correct b/c Compensation means seeking success in an area of life as a substitute for lack of success in another area. Rationalization (answer D) is not correct b/c Rationalization refers to substituting a more socially acceptable reason for an action rather than identifying the real motivation.

A mother and a daughter are meeting with a social worker regarding difficulties they are having in their relationship. The daughter is 16 years old and has dated several boys during the last three months. Her mother is against her dating at this age and is extremely critical of her behavior. The mother states that she is sure her daughter will be getting into trouble. When the mother was asked about her teenage years, the mother stated that she became pregnant at 16. The mother's response to her daughter's dating could be termed A. Regression B. Projection C. Sublimation D. Denail

B- Projection. Projection is a defense mechanism that involves ascribing one's own thoughts, attitudes, or behavior's to someone else. Regression (answer A) is not correct b/c regression is a defense mechanism in which the individual reverts to more primitive modes of coping associated with earlier and safer developmental periods. Sublimation (answer C) is not correct b/c sublimation is a mechanism by which intolerance drives or desires are diverted into activities which are acceptable. Denial (answer D) is not correct as denial is a refusal to acknowledge an aspect of reality, including one's experience, b/c to do so would result in overwhelming anxiety.

A school social worker has been seeing an Asian high school student who has joined a gang of Hispanic youth and together they have participated in criminal activity. The student is dating Hispanic girls and hangs spends time with Hispanic friends. He has told his social worker that he secretly dislikes Hispanics but that he is friendly with them b/c he is lonely. The student is MOST LIKELY using which defense mechansism? A. Denial B. Reaction Formation C. Projection D. Sublimation

B- Reaction Formation. Reaction formation is a defense mechanism that is the antithesis of the instinctual urge (e.g acting as if one has deep sympathies for a certain group when the individual actually has significant prejudices against that group).

Social workers can work as managers in human service organizations at all levels of administration. The primary function of management is to A. Attend to the staffing of the organization B. Plan organizational activities C. Achieve the goals of the organization D. Maintain the moral of employees

C- Achieve the goals of the organization, b/c it is the most general of the responses (i.e the other tasks can be subsumed under "achieve the goals of the organization").

In an initial family therapy session, the two younger children (7 and 10 years old) begin acting out and fighting with each another. The parents do nothing to end the commotion. The social worker should FIRST A. Tell the parents to control their children B. Stop the arguing himself C. Allow the acting out to continue so he can asses the family dynamics D. Teach the parents how to stop the arguing

C- Allow the acting out to continue so he can assess the family dynamics. It is important to note that this is an initial family therapy session, and important time for the social worker to observe the interactions of family members. If this were a later session, the social worker might stop the arguing (answer B) as a means of modeling appropriate behavior for the parents, or he might teach the parents through some other means how to stop the arguing (answer D). Telling the parents to control their children (answer A) is not correct as this does not educate the parents about proper parenting strategies.

A client has recently been hospitalized with delusions and auditory hallucinations. Her speech patterns are normal and her affect is appropriate. Her husband reports that the symptoms started approximately five days ago. Which of the following diagnoses BEST fits the client's symptoms? A. Schizoprenia B. Schizoaffective Disorder C. Brief Psychotic Disorder D. Schizophreniform Disorder

C- Brief Psychotic Disorder. Schizophrenia (answer A) is not correct b/c the client's symptoms would have had to be present for at least 6 months to warrant a diagnosis of Schizophrenia. Schizoaffective Disorder (answer B) is not correct b/c a diagnosis of Schizoaffective Disorder would require a co-occurring depressive, manic, or mixed episode. Schizophreniform Disorder (answer D) is not current b/c the client's symptoms would have had to be present for at least 1 month but less than 6 months in order for a diagnosis to be Schizopreniform.

A 30- year- old woman appears at a family service agency as a walk- in. She has just come from the emergency room where she received care for a broken rib and badly beaten face. She states that she has been beaten by her husband but does not want to go to a shelter since her children are due home from school in an hour and she wants to be home with them. What is the FIRST thing the social worker should do? A. Contact the police. B. Aid the woman in going home to get her children. C. Have the child picked up and get the women to a shelter D. Take hit woman to the shelter.

C- Have the children picked up and get the woman to a shelter. This is the best option b/c it provides adequate protection for both mother and children. Contacting the police (answer A) is not the best FIRST answer b/c the woman and child need to be protected immediately, and then police involvement may occur. Aiding the women in going home to get her children (answer B) is not correct b/c the abusive husband may be at home. Taking the woman to the shelter (answer D) is not correct b/c the mother want to make sure that her children are safe before she makes other plans.

While working with a family during a family session, the mother states that she is so angry that she could hit her children senseless. This comment alerts the social worker to the fact that there may be child abuse in the family. The social worker's NEXT step needs to be to A. Ask the mother if she has hit the children B. Change the subject C. Ask the children if she has hit them before D. Draw the children out to discuss this

A- Ask the mother if she has hit the children. This would be the NEXT logical thing to do, as it was the mother's comments that raised the issue of possible child abuse. Changing the subject (answer B) is not correct b/c when a client reveals information such as that in the scenario, it is never appropriate to change the subject and not deal with the information. If the mother denies that she has hit the children, the social worker may want to further explore the possibility of abuse by asking the children if she has hit them before (answer C) or by drawing the children out to discuss the situation (answer D).

A client reports to her social worker that her 23- year- old friend was raped the night before. The social worker's FIRST responsibility is to A. Assist the client in talking about her thoughts and feelings relative to the rape B. Encourage the client to report the rape to the police C. Report the rape to the police D. Instruct the client to encourage the rape victim to make a police report

A- Assist the client in talking about her thoughts and feelings relative to the rape. If the victim were a child, by law a social worker would be responsible to report the crime to the proper authorities. Encourage the client to report the rape to the police (answer B) may not be appropriate b/c the client does not have all the necessary information. Reporting the rape to the police (answer C) is not correct b/c the social worker would be reporting hearsay information. Instructing the client to encourage the rape victim to make a police report (answer D) is not the best answer, but if the client feels that she should encourage the rape victim to make a police report the social worker should support her in that action.

A 30- year- old unmarried females is eyeing a social worker in private practice to address her anxiety issues. The client feels anxious in social situations and has dated very little, feeling that she will be ridiculed because of her shyness. The client was raised in an abusive family and believes that she is inferior to others. She is excessively sensitive to criticism from her siblings. If she misinterprets a comment from a friend she worries about it until she makes herself sick. The social worker MOST LIKELY will diagnose the client with which of the following conditions? A. Avoidance Personality Disorder B. Social. Anxiety Disorder C. Generalized Anxiety Disorder D. Dependent Personality Disorder

A- Avoidant Personality Disorder. Social Anxiety Disorder (answer B) is not correct as this disorder is evident when performance in some social situations results in anxiety. Avoidance behavior is often evident in this disorder, but Avoidant Personality Disorder also includes social inhibition, low self- esteem and excessive sensitivity to criticism, which makes that the more appropriate diagnosis. Generalized Anxiety Disorder (answer C) is. Not correct as this disorder involves excessive anxiety and worry about a number of things that persists for a minimum of 6 months. Dependent Personality Disorder (answer D) is not correct as this disorder is characterized by an extreme need to be taken care of which leads to submissive behavior and clinginess as well as fears of separation.

A newly- graduated social worker has taken employment in a Youth Corrections agency as an intake worker. She must gather information from the client's and then assign them to other caseworkers. A type of question that is useful in obtaining specific information is A. Closed - ended questions B. Open- ended questions C. Leading questions D. Specific questions

A- Closed ended questions. Closed- ended questions such as "When did you move to Utah? Allow a person to quickly gain specific information. Open- ended questions (answer B) is not correct b/c open- ended questions invite free, as opposed to specific, expression. Leading questions (answer C) is not correct b/c leading questions are designed to elicit agreement with the social worker and are not appropriately used in practice. Specific questions (answer D) is not correct b/c "specific questions" is not a category of questions at all.

"Questioning the evidence" is a technique used in A. Cognitive Therapy B. Behavior Therapy C. Family Therapy D. Transactional Analysis

A- Cognitive Therapy. In characterizing Aaron Beck's Cognitive Tehrapy, the pt's problems are continually defined in cognitive terms. In an attempt to reverse thinking errors, the social worker and pt work together, using a series of questioning techniques to discover reality. Behavior Therapy (answer B) is not correct b/c behavior therapy is a structured approach used to modify an individual's behavior. Family Therapy (answer C) is not correct b/c family therapy focuses on the relationships among family members. Transactional Analysis (answer D) is not correct b/c Transactional Analysis involves an examination of the interactions b/w the ego states of two indivduals.

An Asian couple comes to speak with a social worker regarding their son, age 15, who has been acting out at school and at home. The couple gives only brief answers to the social worker's questions. The MOST LIKELY reason for this is A. Cultural differences B. Language barrier C. Their embarrassment D. Reluctance to talk with someone who is not family

A- Cultural differences. The behavior of clients in counseling situations is often a function of their culture. Language barrier (answer B) is not the best answer for this question b/c although language barrier is a possibility, the most likely answer is cultural differences, and it is a factor that should always be considered. Their embarrassment (answer C) is not correct, as there is no evidence to conclude that they are embarrassed. Reluctance to talk with someone who is not family (answer D) is not a correct answer, as the couples' reluctance to talk with someone who is not family may be characteristic of their culture.

Step- families often deal with a variety of difficult issues. The most common issue in step- families and therefore an important issue for the social worker to be aware of is A. Discipline styles B. Age of parents C. Age of children D. Personalities.

A- Discipline styles. Step- parents often have different discipline styles, and these differences often create conflict in the home. Age of parents (answer B), age of children (answer C), and personalities (answer D) are all not correct answers b/c ages and personalities of parents and children are typically not nearly as important as discipline styles in step- families.

A social worker is seeing a client who is in the advanced stages of HIV. His health has deteriorated to the point that he is no longer able to care for himself. He has been estranged from his parents for years and has not them that he is gay. The client work like to reach out to his parents but does not know how. What should the social worker do FIRST? A. Encourage the client to contact his parents. B. Offer to contact the parents and meet with them separately. C. Encourage the client to not contact his parents as the confrontation would create stress that could worsen his condition further. D. Arrange a meeting between the client and his parents.

A- Encourage the client to contact his parents. Both the social worker and client should be aware that his will be a difficult situation and the social worker should be there to support the client in the process. The client needs to be the one to make contact with his parents. The social worker offering to contact the parents and meet with them separately (answer B) is not correct as the parents should not be in a situation where they may be able to obtain confidential information from the social worker regarding the client. Encouraging the client to not contact his parents as the confrontation would create stress that could worsen his condition further (answer C) is not correct b/c the client has indicated his desire to contact his parents. Arranging a meeting b/w the client and his parents (answer D) is not the best answer since it would not be the FIRST action to take. Although this could be an option suggested to the client or a good "NEXT" step.

A hospital social worker is working with an elderly woman who has broken her hip and is unable to care for herself in her home. The social worker is attempting to arrange a placement for the woman in a skilled nursing center. The woman states that she does not want to go anywhere but to her home. However, there is no one to care for her at home and the services she qualifies for are not adequate to support her in her home. What should the social worker do? A. Explore all possible alternatives B. Insist that she be transferred to a nursing home C. Arrange her her to stay longer in the hospital to work through her concerns D. Talk with her doctor

A- Explore all possible alternatives. The social worker has a responsibility to assist the woman in identity a viable care option. Insisting that she be transferred to a nursing home (answer B) is not correct b/c it would violate the client's right to self- determination. Arranging for the client to stay longer in the hospital to work through her concerns (answer C) is not correct as the social worker may not be able to do this b/c of hospital regulations. Talking with the doctor (answer D) is not correct b/c, although the social worker may talk with the client's doctor to obtain any additional information,, it is the social worker's job to make arrangements for the client.

A young woman seeks a social worker's help in alleviating stress from her life. She is underweight and was diagnosed with Grave's disease several months ago. She states that she is experiencing nervousness in her job and is having a difficult time concentrating. The social worker's FIRST step would be to A. Have the woman speak with her doctor B. Speak with the woman's family C. Work out a behavioral program D. Have the woman's medication increased

A- Have the woman speak with her doctor. Whatever there is a possibility that a medical condition could be creating the symptoms, the patient should be referred to a medical doctor. Once the medical situation is addressed, the other answer options- speaking with the woman's family (answer B), working out a behavioral program (answer C), and having the woman's medication increased (answer D)- may be appropriate courses of action to take. This question asks for the FIRST step that the social worker should take.

A 19- year- old man is referred to a private practitioner by his parents for difficulties maintaining a job. HIs parents have threatened to throw him out of the house. He passively resists doing what is requested of him. When the social worker asks him about having issues with authority, he remains silent and makes not acknowledgment. The social worker should A. Identify this pattern and help the client explore his behavior B. Set up a family therapy session C. Send him to Vocational rehabilitation D. Provide him with a referral to a career counselor

A- Identify this pattern and hep the client explore his behavior. Setting up a family therapy session (answer B) is not correct since the client is an adult and a family therapy session would be a better way of dealing with his problems if he were a child. Sending him to a Vocational rehabilitation (answer C) and providing him with a referral to a career counselor (answer D) are both incorrect, as they are premature steps to take and the client would unlikely be able to benefit from these services at this point in treatment.

During a play therapy session with a 6- year- old child who has been sexually abused, the child begins to rub his genital area against the social worker's leg. The LEAST appropriate action for the social worker to take would be to A. Ignore the behavior B. Disengage the child C. Discuss with the child what is happening D. Ensure this is listed in the case report

A- Ignore the behavior. The question asks for the LEAST appropriate action. Behavior's of the type should never be ignored. They are often indicative of sexual abuse and, at a minimum, need to be interrupted. All other answer options are appropriate courses of action to take in a case such as this.

A social worker in a psychiatric setting in working with client who exhibits characteristics of Bipolar I Disorder. The psychiatrist in the clinic plans to prescribe medication for the client. Which of the following is the most commonly used medication for Bipolar I Disorder? A. Lithium B. Ritalin C. Zoloft D. Prozac

A- Lithium. Ritalin (answer B) is not correct b/c Ritalin is a common mediation used for ADHD. Zoloft (answer C) and Prozac (answer D) are both incorrect b/c they are medications commonly used to treat depression.

A social worker is seeing a husband and wife in couples therapy. The husband has agreed to participate although he believes that their problems are minimal and not worthy of the cost of treatment. In the session the husband attempts to monopolize the therapy hour by telling stories of all the people that he and his wife have met in their travels. He shows the social worker photos of them with famous people and discloses how well he was treated by these people. He believes that his wife does not appreciate all of the 5 star hotels they have stayed in and the lavish parties they have attended. The wife states that she would prefer attention from him and a close relationship with him. Which of the following diagnosis is appropriate for the husband? A. Narcissistic Personality Disorder B. Borderline Personality Disorder C. Histroinic Personality Disorder D. None of the above

A- Narcissistic Personality Disorder focuses on self and wants admiration. Borderline Personality Disorder (answer B) is not correct as this disorder is characterized by instability in relationships, self- image and affect, as well as impulsive bheavior, fear of abandonment, and repeated suicidal threats. Histrionic Personality Disorder (answer C) is not correct as this disorder is characterized by high emotionality and attention- seeking behavior. Both Borderline Personality Disorder and Histrionic Personality Disorder are more common in females, while Narcissistic Personality Disorder is more common in males.

During an intake interview at a mental health center a 22-year-old client who is being evaluated as a suicide risk leaves the office where the evaluation is taking place and walk out the door. The social worker's FiRST course of action is to A. Notify the police B. Notify the client's family if known C. Try to reach the client later that day so as to give him time to adjust D. Wait until the client ties to contact the social worker, as a means to establishing boundaries

A- Notify the police. The question asks what the social worker's FIRST course of action should be. She may notify the client's family (answer B) and she may try to reach the client later in the day (answer C). However, she must first take action that has the great potential to keep the client from committing suicide. Of the options provided, that would be to notify the police. Waiting until the client tries to contact the social worker, as a means of establishing boundaries (answer D) is not correct as a suicidal client may not make an effort to contact the social worker, and if the social worker waits for the client to contact her, it may be too late.

A man and his wife have started marital therapy at her insistence. The wife complains to the social worker that her husband is a perfectionist, controlling, stingy, and a workaholic. Based on the wife's description of her husband's behavior, the social worker would likely diagnose the husband with A. Obsessive- Compulsive Personality Disorder (OCPD) B. Obsessive- Compulsive Disorder (OCD) C. Schizotypal Personality Disorder D. Narcissistic Personality Disorder

A- Obsessive- Compulsive Personality Disorder (OCPD). It is important to be able to distinguish b/w OCD and OCPD. Obsessive- Compulsive Disorder (answer B) is characterized by either obsession or compulsions wheats Obsessive- Compulsive Personality Disorder is characterized by a preoccupation with order, control in relationships, perfection sims, and inflexibility. Schizotypal Personality Disorder (answer C) is not correct b/c Schizotypal Personality Disorder is characterized by a pattern of deficits in interpersonal skills and decrease capacity for close relationships. Narcissistic Personality Disorder (answer D) is not correct b/c Narcissistic Personality Disorder is characterized by grandiosity, a need to be admired by others, and an absence of empathy.

A 7- year- old child is refusing to sleep in his own bed at night. When the parents sit with him in his room, he is fine. When they leave the room, he begins to cry. When they come back, he stops crying. This is an example of A. Operant conditioning B. Classical conditioning C. Punishment cycles D. Higher order conditioning

A- Operant conditioning Operant conditioning occurs when behavior is followed by a reinforcer or when reinforcement is witheheld. In the scenario above, the child's difficulties at bedtime are being reinforced by the parents' response to the child's tears. The child's tears are reinforced by the parent's return to sit with the child. Classical conditioning (answer B) is not correct b/c classical conditioning entails conditioning an individual to associate pleasant feelings with a stimulus that has been anxiety- producing (e.g learning to feel relaxed rather than anxious when flying). Punishment cycles (answer C) is not a correct answer as the child in this case is not being punished. Higher order conditioning (answer D) is not correct b/c higher order conditioning involves paring a neutral stimulus with a conditioned stimulus to produce the same response as the conditioned stimulus.

An important factor to be considered when assessing risk for child maltreatment is A. Parental personality B. Religiosity of parents C. Neighborhood D. Traditional vs. nontraditional marriage

A- Parental personality. There is evidence in the research that certain personality characteristics (e.g low tolerance of frustration or low capacity for empathy) are associated with child maltreatment. Religiosity of parents (answer B) is not correct b/c this is typically not a factor on its own, unless there is information to suggest that this is the case. Neighborhood (answer C) is not correct b/c, while the neighborhood environment can be a factor, the parental personality is more important than the neighborhood. Traditional vs. nontraditional marriage (answer D) is not correct b/c there is no research to indicate that the type of marriage (traditional vs. nontraditional) would contribute to child maltreatment.

In a private adoption agency, a lesbian couple completes an application to adopt a child. The new caseworker begins a vile but is later told by her supervisor that this is not a approved family structure for placing children. The social worker should FIRST A.. Refer the couple to another agency B. Talk with the he could about filing a complaint against the agency C. File a complain with the NASW D. Quit her job

A- Refer the couple to another agency. The question asks for the FIRST thing the social worker should do. Although the social worker may ultimately take other action such as talking with the couple about filing a complaint against the agency (answer. B), filing a complaint with NASW (answer C), and quitting her job (answer D) her first responsibility is to assist the couple in achieving their goal with minimum delay. This would most likely be accomplished by locating an adoption agency that does not restrict lesbian couples from adopting.

In most casework interventions, the most important stage is A. Relationship building B. Implementation C. Assessment D. Termination

A- Relationship building. Mental health research indicates that this is the most important factors in successful outcomes in therapy. Accurate assessment (answer C) is necessary to determine the best treatment plan, and the implementation (answer B) of the treatment by a skillful social worker is a desired goal; however, those answers are incorrect as the successful outcome of therapy is still dependent upon the client's relationship with his or her social worker. Termination (answer D) is not correct as termination is a stage that occurs after the therapy has been successful.

While working at a child welfare agency, a social worker learns that agency staff members are making decisions based upon positive feelings toward certain clients. These feelings may influence staff members decisions regarding financial assistance. The social worker's FIRST step should be to A. Speak with the director and clarify the process B. Organize client's around the issue C. Hold a meeting with all social workers D. Contact the local NASW

A- Speak with the director and clarify the process. The question asks for the FIRST step the social worker should take in an attempt to resolve the issue. When there is an agency- based problem, the initial effort toward resolving the problem should be to discuss the problem with the he director. Organizing client's around an agency practice (answer B) is incorrect b/c that would not be an appropriate activity. Holding a meeting with all social workers (answer C), and contracting the local NASW (answer D) may be necessary as social action steps, but that will depend on the outcome of the meeting with the director.

In counseling with a client at a correctional center who is coming up for parole, it becomes clear to the social worker that the client plans to continue assaulting women. The social worker is asked for his professional opinion regarding the client's parole. The social worker should A. State his concern about the client's intention to continue assaulting women B. Ask for more time to evaluate the client C. Not discuss the case b/c this is confidential information D. Get a no-assault contract from the client

A- State his concern about the client's intention to continue assaulting women. Social workers are ethically bound to be honest in their evaluations of clients. Asking for more time to evaluate the client (answer B) is not correct b/c the question states that the client's intention to continue assaulting women is clear, so the social worker does not need additional time to evaluate the client. Not discussing the case b/c this is confidential information (answer C) is not correct b/c the social worker has been asked for his professional opinion. Getting a no- assault contract from the client (answer D) is not correct b/c that would be of no use in this situation, since the client has clearly stated his plans to continue assaulting women.

A charting system used by many social workers to document their work with clients is referred to as SOAP. What does the acronym SOAP stand for? A. Subjective, objective, assessment, plan B. Subjective, objective, analysis, plan C. Subjective, objective, assessment, phase D. Subjective, objective, analysis, phase

A- Subjective, objective, assessment, plan. SOAP is a charting system used frequently by healthcare providers. The "S" stands for subjective information or that information reported by the client or significant others; the "O" stands for objective information such as scores on psychological tests; the "A" stands for assessment, or in other words, the conclusions derived from data; and the "P" stands for plan as in the plan for problem resolution.

A social worker sees a client walking toward her in the grocery store. The social worker should A. Take her cue from the client B. Smile but say nothing C. Say hello D. Turn and go the other way

A- Take her cue from the client. Ideally, the social worker would have determined beforehand what the client wants the social worker to do if they encounter each other in a public place. If she has not done this, she can determine who best to respond to the individual client by observing how he or she responds to the encounter. Smiling and saying mother (answer B) and saying hello (answer C) are socially appropriate responses even when encountering a stranger, but these actions may be uncomfortable for the client. Turning and going the other way (answer D) is not correct b/c the client may misinterpret the social worker's response and feel rejected.

Cognitive restructuring is being applied to a client in an outpatient session. The client has a diagnosis of marital discord and depression. While performing the technique, the client becomes very agitated. The best course of action would be to A. Temporarily pause the technique B. Continue the technique C. Lower the agitation D. Focus on positives of technique

A- Temporarily pause the technique. Cognitive restructuring is when one learns to replace disparaging remarks with positive messages. If the process of cognitive restructuring elicits a strong emotional response, the process needs to be temporarily discontinued while the social worker and the client identify the belief that gave rise to the emotions. When this has been accomplished, the process of cognitive restructuring can be resumed. Continuing the technique (answer B) is not correct b/c to do so would likely continue the agitation. Te partly pausing the technique (answer A) would be the action that would lower the agitation (answer C). The social worker can then discuss with the client the positive benefits of cognitive restructuring (answer D).

In using Anatabuse to treat alcohol abuse, the discontinuation of the drug should occur when A. The client utilizes new skills for abstinence B. The client says that he can stay away from alcohol on his own C. The drug makes the client ill D. The client's use of alcohol decreases

A- The client utilizes new skills for abstinence. The client saying he can stay away from alcohol on his own (answer B) is not a correct answer b/c he need to learn new skills to find a replacement for the alcohol use, and it is difficult for alcoholics to stay away from alcohol on their own. The drug making the client ill (answer C) is not correct b/c the purpose of Antabuse is to make the client ill if he uses alcohol. Since Antabuse is a powerful drug, it cannot be used while the alcoholic is still drinking any amount. The client's use of alcohol decreasing (answer D) is not correct b/c the client must refrain from using alcohol altogether rather than simply decreasing his use.

A social worker has been seeing a client- who is an accomplished artist- in private practice for ten months. The client has made excellent progress and is ready to terminate therapy. During the final session the client presents the social worker with apt picture of his dog, which she painted from a photograph that he has on his desk. What is the BEST way for the social worker to handle this situation? A. The social worker should explain that he cannot accept this give, even though this is a kind and generous gesture B. The social worker should remind the client that he discussed gift giving during this initial interview C. The social worker should offer to but the painting from the client D. The social worker should accept the giver as it is a painting of his dog and would not have meaning to anyone else

A- The social worker should explain that he cannot accept the gift, even though this is a kind and generous gesture. Since this client is a professional artist, this fight has significant monetary value, and, subsequently, there is possibility that the client's family could bring a lawsuit against the social worker for accepting a gift of this value. In general, it is not good practice for social workers to accept gifts from clients. Reminding the client that the social worker discussed gift giving during the initial interview (answer B) may be part of the explanation to the client, but first the social worker should make it clear that is unethical for him to accept the gift. Offering to buy the painting from the client (answer C) is not correct b/c this action would result in a conflict of interest with this client. Accepting the gift as it is a painting of the social worker's dog and would not have meaning to anyone else (answer D) is not correct b/c, even though a painting of his dog would have more meaning to him, it very well could be sold to another individual.

During a session with a client, a social worker responds to a client's angry outburst with, "I feel upset when you raise you voice at me". The social worker is utilizing A use of self B. Empathy C. Self- determination D. Summarizing

A- Use of self. The social worker has disclosed something about herself to promote growth in the client. This constitutes use of self. Empathy (answer B) is not correct b/c empathy is the action of accurately reflecting the surface feelings of the client. Self- determination (answer C) is not correct as self- determination refers to the client's right to make his or her own decisions. Summarizing (answer D) is not correct b/c summarizing provides the client's with a condensed version of a segment of an interview.

A teenage drug abuse group becomes hostile and the social worker fears that violence will break out. It is important for the social worker FIRST A. Try to de-escalate the group B. Ask the two most difficult members to leave C. Call for help D. End the group

A- try to de- escalate the group. This question asks what the social worker should do FIRST. Asking the two most difficult members to leave (answer B) may be a necessary step in de- escalating the group, but is not the first step. If the social worker is unsuccessful in efforts to de-escalate the group, he may have to call for help (answer C) or end the group (answer D).

While working with an elderly patient on a home- health basis, the social worker becomes aware that she has developed numerous bruises on her legs. When the social worker asks about these bruises, the patient states that she often hits her legs with her walker when she is trying to get around. The social worker knows that she has used this Walker for almost two years and has not has this problem before. The social worker is also aware that her daughter, who is her sole caretaker, has appeared very stressed out when she has run into her at her mother's home. The social worker needs to A. Have the walker examined B. Assess for elder abuse C. Accept the client's explanation D. Have a private meeting with the daughter

B- Assess for elder Abuse. It is unlikely that the pt has suddenly started having difficulties using the walker. The sudden appearance of brushing on the pt's legs in the absence of a reasonable explanation for the bruises, in combination with the emotional condition of the women's caretaker, suggests the need to assess for elder abuse. Having the walker examined (answer A) could be part of the assessment for elder abuse, as could having a private meeting with the daughter (answer D). Accepting the client's explanation (answer C) is not correct b/c the social worker can see the client's explanation is not likely at this point.

A social worker has been working with a family that has a 16- year-old son who has had a drug problem. The son, in an individual session, tells the social worker that he has stated drug use again. The social worker's FIRST responsibility is to A. Maintain the son's confidentiality B. Assesses the son's drug use C. Disclose this information to his parents D. Ask the son to identify the reasons he has started using again

B- Assess the son's drug use. The question asks about the social worker's FIRST responsiblity. Before the social worker takes any action, it is important to make a careful assessment of the sone's current drug use. Maintaining the son's confidentiality (answer A) is not the best answer as the question asks for the social worker's FIRST responsibility in this situation, and that is to find out the facts about the drug use. Disclosing the information to his parents (answer C) is not the best answer b/c the social worker needs to assess the young man's drug use before taking any further action. Asking the son the reasons he has started using again (answer D) is not the best answer as this may be a step to be taken as part of assessing the son's drug use.

During a session with a client, the social worker loses track of conversation while day-dreaming. The client asks the social worker if she is paying attention. The social worker should A. Pretend she was thinking B. Be honest about what happened C. Talk with the client about her reasons for asking D. End the session

B- Be honest about what happened. The quality of the therapeutic relationships is a function, in large part, of the extent to which the social worker responds to the client In an honest and forthright manner. Pretending that she was thinking (answer A) is not correct b/c for the social worker to pretend that she was thinking would be an insult to the client, as the client obviously knew what was going on. Talking with the client about her reasons for asking (answer C) and ending the session (answer D) are not correct b/c both are ways of diverting the issue.

While discussing a relationship problem in an initial session of couples counseling, the husband begins to talk about his wife's sexual difficulties. The wife becomes tearful and silent. The social worker's response to this situation is best handled by A. Encouraging the wife to talk B. Being empathetic to the wife's position C. Setting up an individual session D. Allowing the conversation to continue uninterrupted

B- Being empathetic to the wife's position. Empathy is a key component of a therapeutic relationship. A demonstration of empathy for the feelings that the husband's comments have elicited in the wife can both strengthen the therapeutic relationship and provide a safe environment to discuss sensitive issues. Encouraging the wife to talk (answer A) is not correct b/c this step should occur at a later time, after the therapeutic relationship has been firmly established, rather than during the initial session. Setting up an individual session (answer C) is not the best answer b/c this may or may not be an appropriate strategy, depending on the circumstances. Allowing the conversation to continue uninterrupted (answer D) is not correct b/c this approach could be viewed as support of the husband.

A social worker has been seeing a 60- year- old client for depression for several months. The client recently underwent laser resurfacing on her face to diminish wrinkles. After she recovered from the procedure she had additional procedures performed, including Botox and fillers for lines around her mouth and eyes. Six months later she had an operation to reduce the size of her nose and two months after that had surgery to remove the excess skin on her eyelids. She has had treatments to reduce discoloration of her skin and to enlarge her lips. She is now considering having a surgical facelift to completely eliminate sagging and wrinkled skin. After all of these procedures, she is still depressed about her appearance. What is the likely co-occurring diagnosis for this patient? A. Somatic Symptom Disorder B. Body Dysmorphic Disorder C. Adjustment Disorder with Depressed Mood D. None of the above

B- Body Dysmorphic Disorder, which is characterized by a preoccupation with a slight or imagined physical defect that the individual believes negatively affects his or her appearance. In this case, the natural body changes associated with age have created a significant problem for this client, which is interfering with her ability to function. Somatic Symptom Disorder (answer A) is not correct as this is a condition characterized by the client's fear that he or she has a series medical condition. Adjustment Disorder with Depressed Mood (answer C) is not correct as an Adjustment Disorder is characterized by the development of emotional/ behavioral symptoms within 3 months of a stressor, and there is not evidence that this client has had a significant stressor in her life within the past three months.

A social worker is treating a client for test anxiety. The client comes in to see the social worker shortly after taking an algebra examination. When the social worker asks her how the test went, the client responds, "Awful, I just can't do anything right." According to Beck, this is an example of which common thinking error? A. Personalization B. Catastrophizing C. Mind reading D. Minimizing

B- Catastrophizing. Catastrophizing is assuming the worst or overemphasizing the negative. The client is engaging in Catastrophizing when she states that she cannot do anything right. Personalization (answer A) is not correct b/c personalization is a thinking error that occurs when an individual assumes that someone else's negative behavior is the result of something that he or she has done. Mind reading (answer C) is not correct as mind reading is thinking error that occurs by an individual assuming that he or she knows another person's thoughts about something. Minimization (answer D) is not correct b/c minimization occurs when a person magnifies the negative and minimizes the positive in evaluation.

A social worker in private practice has been working with a male client who is a successful businessman in his community. The client is not making progress in therapy and has a habit of blaming others for his relationship problems with his family. A method of helping this client become more self- aware by brining to light components of high thoughts, feelings, or behavior of which he is unaware is called A. Summarizing. B. Confrontation C. Active listening D. Reflective listening

B- Confrontation. Summarizing (answer A) is not correct b/c summarizing is a verbal following skill which provides the client with a condensed version of a segment of an interview. Active listening (answer C) is not correct b/c active listening is a set of listening skills used in problem resolution, wherein the listener rephrased the message (verbal and nonverbal components) sent by the communicator. Reflective listening (answer D) is not correct b/c reflective listening is another term for active listening.

In a session a client states that he is very angry with his ex- wife and he plans to "let her have it". His social worker asks him what "let her have it" means, and he replies, "It means that she isn't going to be here anymore". The social worker's ethical responsibility is to A. Inform the client that the social worker will need to call the police B. Contact the ex-wife if her information is available and also contact the police C. Set up a no-harm contract D. Contact security

B- Contact the ex- wife if her information is available and also contact the police. In Tarasoff v. University of California Board of Regents, the court determined that mental health providers have a legal responsibility to not only inform the police of a client's intent to harm another person but to warn the intended victim, as well. Informing the client that the social worker will need to call the police (answer A) is not the best answer b/c the intended victim and the police must be notified whether the client is advised or not. Setting up a no- harm contract (answer C) is not correct b/c setting up such a contract does not meet the social worker's legal responsiblity. Contacting security (answer D) is not correct b/c contacting security is insufficient warning to the intended victim.

A recently divorced 45- year- old woman has sought therapy because she is having a difficult time adjusting to her divorce. The woman feels frightened and alone and has begun online dating. She would like to remarry because she believes that she cannot take care of herself. IN the therapy sessions the client needs constant reassurance from her social worker that she is making good decisions. The client is passive in the therapy sessions and usually agrees with whatever the social worker suggests. It is obvious to the social worker that the client fears losing her approval and support. What is the MOST LIKELY diagnosis for this client? A. Avoidant Personality Disorder. B. Dependent Personality Disorder C. Passive Personality Disorder D. Social Anxiety Disorder

B- Dependent Personality Disorder. Avoidant Personality Disorder (answer A) is not correct as this disorder is characterized by social inhibition, low self- esteem, and excessive sensitivity to criticism. Passive Personality Disorder (answer C) is not correct as this is not a DSM-5 mental health disorder. Social Anxiety. Disorder (answer D) is not correct as social anxiety disorder is evident when performance in some social situations results in anxiety and avoidance behavior is often evident.

A 36- year- old man experiencing sexual dysfunction comes to a social worker for help. He is not married and has recently been dating several woman. He states that he has been sexually active with all of them. FIRST and foremost, the social worker needs to A. Determine whether his sexual partners have included men B. Determine whether he is having difficulties with all the woman C. Determine whether the women are experiencing sexual difficulties D. Speak with the woman

B- Determine whether he is having difficulties with all the woman. The question asks the social worker to identify the FIRST thing he or she needs to do with this client. Therapy starts out with an assessment of the problem the pt is experiencing. As part of an assessment of the client's problem, it is important to determine whether he is having similar problems with all of the women. This will be an important factor in achieving an accurate understanding of the client's problem. Determining whether his sexual partners have included men (answer A) is not correct as the client has not mentioned men as sexual partners, only women. Determine whether the women are experiencing sexual difficulties (answer C) and speaking with the women (answer D) are not correct b/c it is the client who has asked you for help, not the women.

A social work supervisor learns that an employee has not followed through on a directive she gave her regarding a client. The supervisor should A. Fire the employee B. Discuss the issue with the employee C. Transfer the employee D. Report the employee to Human Resources

B- Discuss the issue with the employee. The social worker may ultimately have to take other action such as firing the employee (answer A), transferring the employee (answer C), or reporting the employee to Human Resources (answer D), but it is important that she first discuss the issue with the employee so she will be able to determine the best course of action to take in the situation.

The court referred a young man to a social worker for harassing women whose shoes, according to his claims, turn him on. The medical term for this is A. Exhibitionistic Disorder B. Fetishistic Disorder C. Frotteristic Disorder D. Transvestic Disorder

B- Fetishistic Disorder. According to DSM-5, a Fetish involves intense and recurring urges, fantasies, or behavior's in response to inanimate objects as is the case with the woman's shoes. Exhibitionistic Disorder (answer A) is not correct b/c it is characterized by intense and recurring urges, fantasies, or behavior's that are sexually arousing in relation with exposing one's genitals to people the individual does not know. Frotteristic Disorder (answer C) is not correct b/c it is characterized by intense and recurring urges, fantasies, or behavior's that are sexually arousing and focused on touching or rubbing against a person who has not given consent. Transvestic Disorder (answer D) is not correct b/c it refers to an individual who dresses in the clothing of the opposite sex, and is not considered a mental disorder.

A social worker in a child abuse center creates a group of abusive parents. During the first session, one of the mothers becomes angry and starts to argue with another parent. The social worker should handle this situation by A. Asking the mother to explain what caused her anger B. Having the parents work through the disagreement In an appropriate manner C. Insisting the mother stop arguing D. Allowing the mother to vent her anger

B- Having the parents work through the disagreement In an appropriate manner. Asking the mother to explain what caused her anger (answer A) is not correct b/c what caused the anger is not nearly as important as learning to resolve conflicts in appropriate ways. Insisting that the mother stop arguing (answer C) is not correct b/c this action does not help her to learn appropriate ways to resolving differences. Allowing the mother to vent her anger (answer D) is not correct b/c the tendency of the parents in the group to vent their anger has been previously problematic for them.

During a group session, a client shares that 7 years ago he set a building on fire, which resulted in the death of security guard. The social worker may do all of the following EXCEPT A. Ask group members how they want to handle the situation B. Ignore the statement b/c the situation happened 7 years ago C. Check out members' feelings D. Encourage the client to speak with the police

B- Ignore the statement b/c the situation happened 7 years ago. The question asks which option the social worker should NOT take. A social worker should never ignore a disclosure of this type, no matter when it occurred.

A counselor at a local high school contacts a social worker in a child welfare protection agency and describes the situation of a 14- year- old girl reporting sexual abuse by her father. The social worker's FIRST step should be to A. Remove the child from the home B. Interview the child immediatley C. Contact the police D. Contact the family.

B- Interview the child immediatley. The question asks what the social worker's FIRST step should be. After interviewing the child, the social worker will be Ina position to determine how best to proceed with the case. Removing the child from the home (answer A) is not correct b/c the social worker must first determine the circumstances of the case before taking such drastic action. Contacting the police (answer C) is not correct as thee worker does not yet know whether police need to be involved. Contacting the family (answer D) is not correct b/c the worker must first respond to the school counselor's referral by following agency procedure, which is to interview the child immediately.

A 42- year- old man arrives at a community mental health center with the following symptoms: pressured speech, racing thoughts, and grandiose ideas. Which of the following is apt to be the cause of the symptoms? A. Alcohol use B. Mania C. Major Depressive Disorder D. Somatization

B- Mania. There are common symptoms of a manic episode. Alcohol use (answer A) is not correct, as the symptoms of alcohol use would include slurred speech, unsteady gait, impaired judgement, mood lability, and impaired motor skills. Major Depressive Disorder (answer C) is not correct, as this would be characterized by sadness, hopelessness, and reduced activity. Somatization (answer D) is not correct b/c Somatization symptoms include pain, gastrointestinal problems, sexual symptoms, and pseudoneurological symptoms.

A husband and wife are seeing a social worker for problems in their marriage. The husband sees no need to participate in marriage counseling because he feels that he is contributing to the marriage by earring an adequate income. The wife is unhappy because she feels emotionally abused. Her husband accuses her of having affairs with the pastor of their church, their next door neighbor, the local grocer, and their daughter's boyfriend- all of which are untrue. If the wife attempts to reason with the husband he feels that she is threatening his role as "head of the house". Often he interprets her innocuous remarks as personal attacks and then refuses to speak to her for several days. He refuses to enter into any kind of business arrangement with others because he believes the will take advantage of him. Which of the following diagnoses is the husband exhibiting? A. Schizophrenia B. Paranoid Personality Disorder C. Intermittent Explosive Disorder D. Bipolar I Disorder

B- Paranoid Personality Disorer. He is suspicious and believes that others have malicious intent. Schizophrenia (answer A) is not correct as that disorder is characterized by prominent delusions, typically of the persecutory and grandiose type, disorganized thought, or auditory hallucinations. Intermittent Explosive Disorder (answer C) is not correct as this disorder is characterized by a number of separate episodes in which the individual gives in to aggressive impulses resulting in serious assaults or damages to property. Bipolar I Disorder (answer D) is not correct as this disorder involves a minimum of 1 manic or mixed episode and will usually have Major Depressive Episodes.

A college student has been in counseling for several weeks. She reports having difficulties falling asleep and nightmares related to the rape of her sister that she witnessed four months ago on her school campus. She now misses classes and after school activities. She says that she feels angry, disconnected from others, and has gaps in memory when she talk about the rape of her sister. Which of the following diagnoses best describes the client? A. Anxiety Disorder B. Posttraumatic Stress Disorder C. Adjustment Disorder D. Nightmare Disorder

B- Posttraumatic Stress Disorder. The indicators are more severe than those for Anxiety Disorder (answer A) and Adjustment Disorder (answer C). There is insufficient data to diagnose Nightmare Disorder (answer D).

A 5- year- old male has been in four different foster homes over his lifetime. His current foster parents are concerned about his seeming inability to form relationships with people. He responds to the efforts of others to engage him, including his holster parents, with hostility. Which of the following would be the most appropriate diagnosis for this child? A. Conduct Disorder B. Reactive Attachment Disorder C. Oppositional Defiant Disorder D. Separation Anxiety Disorder

B- Reactive Attachment Disorder. This diagnosis, according to DSM-5, is characterized by "markedly disturbed and developmentally inappropriate social relatedness before age five" frequently resulting from repeated changes of primary caretakers. Conduct Disorder (answer A) is not correct b/c Conduct Disorder involves a pattern of violating the rules and norms of society and the rights of others in a variety of settings. If the onset is before the age of 10 it is called. Childhood Onset Type; onset at age 10 or later is called adolescent onset type. Oppositional Defiant Disorder (answer C) is not correct b/c Oppositional Defiant Disorder is known for the frequent occurrence (over at least a 6 month period) of negativistic, hostile, or defiant behavior that is typically manifested by members of one's peer group. Separation Anxiety. Disorder (answer D) is not correct b/c separation anxiety disorder is distinguished by excessive anxiety around separation from significant others.

A child welfare worker has been assessing custody of a child in a divorce case. The guardian ad litem has also been making this assessment. MOST LIKELY, the guardian ad litem's assessment will be A. Different B. Similar C. Biased D. Formulative

B- Similar. The primary concern of both the child welfare worker and the guardian ad litem is the well-being of the child. In making an assessment with the well-being of the child in mind, both workers use the same information.

There are different styles of leadership that can be utilized in organizations as a function of management. Which of the following describes the laissez-faire style of leadership in organization? A. Staff input on decision in sought, administration makes the final decision B. Staff members have a high level of say in decision-making; administration is only minimally involved in the process C. Administration engages in unilateral decision making D. None of the above

B- Staff have a high level of say in decision making; administration is only minimally involved in the process. When staff input on decisions is sought and administration makes the final decision (answer A) the democratic style of leadership is being used. When administration engages in unilateral decision- making (answer C) the authorization style of leadership is being used.

An experienced social worker has recently been employed in a small family therapy clinic. The social worker has been asked to provide supervision to staff members and she wants to do a good job. She knows that effective supervision is A. Unstructured, consistent, and case- oriented B. Structured, consistent, and evaluated C. Unstructured, consistent, and evaluated D. Unstructured, consistent, and theory- oriented

B- Structured, consistent, and evuated. It is the only answer in which all three components are accurate. The other response options each contain one incorrect component (i.e. Unstructured).

A counter- conditioning intervention frequently used in the treatment of phobias is A. Assertiveness training B. Systematic desensitization C. Sensate focus D. Modeling

B- Systematic desensitization. Assertiveness training (answer A) is not correct b/c assertiveness training involves training an individual to communicate his or her feelings in a direct and honest manner. Sensate focus (answer C) is not correct b/c sensate focus is used in the treatment of sexual performance anxiety. Modeling (answer D) is not correct b/c modeling is a strategy used to show client's how to perform certain behaviors.

A client calls his social worker to discuss an argument he and his wife have had while they have been vacationing in Mexico. The social worker has been seeing the couple together and the wife in unaware of the phone call. The BEST course of action is to A. Continue the conversation with the husband B. Tell the husband that the discussion will need to wait until the couple returns from their vacation so the wife can also participate in the discussion C. Arrange a phone session D. Refuse to speak with the husband

B- Tell the husband that the discussion will need to wait until the couple returns from their vacation so the wife can also participate in the discussion. Couple issues are best dealt with in a conjoint, in- person session. Continuing the conversation with the husband (answer A) is not correct b/c the COUPLE needs to have the conversation, not the social worker and his bad. Arranging a phone session (answer C) is not correct b/c a phone session would not be as productive as an in- person session and is not required at this time. Refusing to speak with the husband (answer D) is not correct as the social worker can explain to the husband the need to wait for the session, rather than refusing to speak to him.

A local counseling center is planning to start a bereavement group for families who have suffered the loss of a loved one to AIDS. The program will have an evaluation component that assesses the program from the planning stage through the implementation stage. The type of evaluation is called A. A summarize program evaluation B. A cost- benefit analysis C. A formative program evaluation D. A cost effectiveness evaluation

C- A formative program evaluation, which assesses a program from the planning stage through the implementation stage. A summarize program evaluation (answer A) is not correct b/c a summarize program evaluation assesses the extent to which a program has achieved its goals. A cost- benefit analysis (answer B) is not correct b/c a cost- benefit analysis compares the costs of a program with its benefits. A cost effectiveness evaluation (answer D) is not correct b/c a cost effectiveness evaluation compares the cost of a program to the program output.

A white social worker has a large proportion of Hispanic clients. The social worker's race frequently comes up as an issue for these clients. The best way for the social worker to handle the issue when it is raised is to A. Change the subject B. Inform the client that the is experienced in working the Hispanic individuals C. Address the issue directly when the client brings it up D. Refer the client's to another social worker

C- Address the issue directly when the client brings it up. Changing the subject (answer A) is incorrect b/c social workers have a responsibility to address concerns client's have about such things as racial or cultural differences. Informing the clients that the social worker is experienced in working with Hispanic individuals (answer B) is incorrect b/c it is too narrow. The issue clients raise about racial or cultural differences are many and varied. Referring the clients to another social worker (answer D) is incorrect b/c frequently when these kinds of issues are death with directly, the client and worker find they are able to work effectively together.

During a group discussion with an outpatient mental health group, one of the members begins to talk in detail about a rape of a young woman he committed several years earlier. The group has agreed to confidentiality as a ground rule. As group members begin to become anxious and regard the member in disbelief, the social worker's FIRST action would be to A. Reinforce the confidentiality rule B. Call the police and report the client. C. Allow group members to discuss their feelings D. Speak with the client individually

C- Allow group members to discuss their feelings. The question asks what the social worker's FIRST action should be in the group. The social worker needs to first deal with what is happening in the her-and-now (i.e. The feelings of the group members). She may then take some other action such as talking with the client individually. Reinforcing the confidentiality rule (answer A) is not the best FIRST action to be taken in this situation, but it is important to reinforce the confidentiality rule with the group before group time is over. Calling the police and reporting the client (answer B) is not the best FIRST action to take in this situation, although calling the police may be a step to be taken after more information is obtained. Speaking with the client individually (answer D) is not a good FIRST course of action in a group therapy sitatuion, as the purpose of a group is to deal with all of the members at the same time, not splinter the group by seeing one member individually.

Which of the following DSM disorders is more commonly diagnosed in males? A. Malingering B. Major Depressive Disorder C. Antisocial Personality Disorder D. Generalized Anxiety Disorder

C- Antisocial Personal Disorder. Research indicates that males are more often diagnosed with Antisocial Personality Disorder than are females, while there is a more even gender distribution in the other disorders listed.

A 15- year- old male reported to his social worker that he recently told his foster care parents that he is gay. The boy is very concerned because the foster care family responded by telling him that he needs to be out of the house by the end of the week. What is the FIRST think that the social worker should do? A. Call the foster care parents to set up a meeting B. Help the boy make other living arrangements C. Ask the boy what he would like to do at this time D. Speak with her (the social worker's) supervisor about the issue

C- Ask the boy what he would like to do at this time. It is important to first understand the client's wishes, thoughts, and feelings before taking action. All of the other answer options may be appropriate AFTER this step is completed, depending on the client's feedback on the issue.

A client has recently been referred to a social worker for issues related to domestic violence and drug use. The client asks questions about the social worker's credentials and experience during the initial assessment. What should the social worker do NEXT? A. Confront the client about how he is avoiding intake questions B.. Discuss the issues related to the domestic violence C. Ask the client how he feels about being in therapy D. Move the conversation to less threatening topic

C- Ask the client how he feels about being in therapy. Since it is the initial assessment, it is too soon to confront the client (answer A). In the other answer options, discussing the issues related to domestic violence (answer B) and moving the conversation to a less threatening topic (answer D), the social worker is avoiding the client's questions. Asking the client a direct question about his feelings is appropriate for the initial assessment and redirects the client back to the reasons for seeking help.

A Malle patient in a psychiatric hospital tells his social worker that a female patient looked at his chart and discovered that he has been diagnosed with Schizophrenia. The male patient is very upset. What should the social worker do FIRST? A. Explain to the client what Schizophrenia means B. Report the other patient for looking in this client's chart C. Ask the client why he is upset D. Let thte client see his chart.

C- Ask the male patient why he is upset. The social worker needs to gather more information and begin where the client is. The social worker should not assume that she know why the client is upset; it could be an entirely different reason from what the social worker thinks. Explaining to the client what Schizophrenia means is not the first step that the social worker should take, and it may not even be necessary depending upon why the client is upset. Reporting the other patient for looking in the client's chart (answer B) is not correct as the first step. The hospital has rules regarding patients' charts and the social worker can look into that after she has talked with her client. Letting the client see his chart (answer D) is not correct as the first step. The social worker may choose to do that later, but not initially.

Rolinda is a 27 year old female who was referred to a social worker for issues regarding depression and anxiety. As the social worker interviewed Rolinda, the client reported that when she feels depressed and anxious that she finds herself "eating more that I should". Rolinda eats when she is not hungry and once she starts eating, she is unable to stop until she does feels uncomfortably full. She has been doing this on average twice a week for the past 6 months. Rolinda feels guilty and disgusted with herself. She appears to be overweight, but she is not obese. The BEST diagnosis that describes Rolinda's behavior is: A. Bulimia Nervosa B. Overeating Disorder C. Binge- Eating Disorder D. Major Depressive Disorder

C- Binge- Eating Disorder. Rolinda meets the criteria for feeling out of control when she eats, feeling uncomfortably full and feeling guilty or embarrassed. Individuals with Binge- Eating Disorder are not necessarily obese. A is not correct. Although Rolinda engages in binges, she does not use compensatory behavior's. B is not correct. There is no DSM-5 diagnosis for Overeating Disorder. D Major Depressive Disorder is also not correct. Although Rolina was referred for depressive symptoms, there is not sufficient evidence presented that justifies this diagnosis.

A client has arrived on her social worker's office for her regular 9:00 am appointments. The client reports feeling "sick" every time that she comes for her appointment and wonders why this occurs. She states that she is not "a morning person" and does not usually get out of bed until 10:00 am. She says that he is experiencing nervousness, restlessness, muscle twitches, and gastrointestinal issues. She looks somewhat flushed and is rambling in her speech. The social worker should FIRST asses the client for which of the following A.. Cannabis use B. Panic attack C. Caffeine intoxication D. Acute stress disorder

C- Caffeine Intoxication. Although the client has not indicated caffeine use, the symptoms described meet the criteria for Caffeine Intoxication and the social worker should asses the possibility of such. The client has also indicated that she is "not a morning person" and is reasonable to ask the client if she is drinking a lot of coffee to increase her energy levels. There is nothing to suggest that the client is using Cannabis (answer A) and she is not presenting with the associated symptoms. While the client is showing some signs of a Panic Attack (answer B) this scenario more accurately describes Caffeine Intoxication. Acute Stress Disorder (answer D) is incorrect b/c there is not indication that the client has experienced a traumatic event and the client's symptoms are not representative of this disorder.

A social worker at a homeless shelter is asked by a mother to take her children to the social worker's home on Christmas Eve so the children will have a Christmas in a nice home. The social worker should A. Take the children home on Christmas Eve B. Explain the ethical reasons why this cannot be done. C. Call the local family service agency to obtain more support for the mother D. Spend Christmas Eve with the family

C- Call the local family service agency to obtain more support for the mother. Taking the children home on Christmas Eve (answer A) is not correct b/c it would be unethical for the social worker to take the children home for Christmas Eve or to spend Christmas Eve with the family (answer D). Explaining the ethical reasons why this cannot be done (answer B) is not the best answer b/c, although it would be appropriate for the social worker to explain this to the mother, the social worker should take action that could assist the mother in gaining access to needed resources.

While working with a client who is experiencing difficulty getting along with the staff where she works, the client and the social worker become aware of a dysfunctional response pattern she has to staff in a specific type of situation. The best course of action to help the client clearly identify what is happening and to change the maladaptive pattern is to use the technique of A. Empty chair B. Mirror C. Cognitive Restructuring D. Role Playing

C- Cognitive restructuring. Cognitive restructuring is when one learns to replace disparaging remarks with positive messages. There is much support in the literature for the value of cognitive restructuring in resolving this type of problem. Empty chair (answer A) is not correct as empty chair is a technique used in Gestalt therapy, whereby the client is asked to act out feelings or thoughts to increase awareness, rather than to learn skills to restructure maladaptive behavior. Mirroring (answer B) is not correct as mirroring is a no directive approach in which the social worker reflects back the client's exact words and gestures. Role playing (answer D) is not the best answer b/c, although role playing may be a useful exercise to practice one's behavior, the cognitive restructuring needs to be the primary therapeutic approach.

A client, who is 16 years old, lost his leg in a serious automobile accident four years ago. Before the accident the client has excelled in sports. Since the accident he has devoted himself to developing computer proficiency. A psychoanalytically oriented social worker would account for this client's post- accident obsession with computers as A. Conversion B. Denial C. Compensation D. Reaction formation

C- Compenstaion. Compensation refers to seeking success in one area of life as a substitute for success in another area of life in which the individual is barred from excelling. Conversion (answer A) is not correct b/c conversion is a defense mechanism by which anxiety is transformed into a physical dysfunction such as a paralysis or blindness that does not have a physiological basis. Denial (answer B) is not correct as denial is a refusal to acknowledge an aspect of reality, including one's experience, b/c to do so would result in overwhelming anxiety. Reaction formation (answer D) is not correct b/c reaction formation is adopting a behavior that is that antithesis of the institutional urge (e.g acting as if one has deep sympathies for an oppressed group when the individual actually has significant prejudices against that group).

A hospital social worker has been assigned to work with a 72- year- old man who wants to go home. He also tells the social worker that his family wants him to stay in the hospital. What is the FIRST thing that the social worker should do? A. Talk with the family abou their reasons for wanting him to stay in the hospital B. Talk to the doctor abou the man's condition C. Explore the client's feelings D. Arrange for the client to be transferred to an assisted living facility

C- Explore the client's feelings. The man is the client and the social worker should "begin where the client is". Talking to the doctor about the man's condition (answer B) is not correct as the FIRST step that the social worker should tak, although it will be an important step in the future. Talking with the family abou their reasons for wanting him to stay in the hospital (answer A) is alos not the first step but is a good NEXT step. Arranging for the client to be transferred to an assisted living facility (answer D) may or may not be a course later on, but is certaintly not the FIRST step before gathering more information abou this case.

A social worker receives a new client who is seeking cognitive behavioral therapy, which is not the social worker's treatment of expertise. The social worker should A. Attempt this intervention B. Refer the client to an expert C. Explore the client's reasons for wanting cognitive behavioral therapy D. Use his own techniques as he or she is well trained to meet the client's needs

C- Explore the client's reasons for wanting cognitive behavioral therapy. As the social worker discusses this with the client, it may become evident that the social worker is able to provide the services the client desires. Alternatively, the discussion may reveal the need to refer the client elsewhere. Attempting this intervention (answer aa) is not correct b/c a social worker should be trained and competent in the interventions he or she uses. Referring the client to an expert (answer B) is not the best answer b/c the social worker should find out more about the client request before referring the client to an expert in cognitive behavioral therapy. Using the social worker's own techniques as he or she is well trained to meet the client's needs (answer D) is not correct b/c the client has asked specifically for cognitive behavioral therapy and should be part of the decision- making process regarding whether other techniques could be as beneficial.

In a hospital setting, a social worker working with an elderly man is instructed by a physician to find the man a nursing home, as the doctor believes that the man is unable to care for himself at home. The man does not want to go to a nursing home, and the social worker is aware of some services that might enable the man to continue to live independently. However, the doctor refuses to listen. The social worker's FIRST responsibility is to A. Follow the hospital's rules B. Follow the doctor's orders C. Follow the patient's wishes D. Follow the social worker's own opinion

C- Follow the patient's wishes Social workers have a primary responsibility to assist their pts in meeting their needs and achieving their goals. Following the hospital's rules (answer A) is not the best answer b/c the pt's wishes should be the primary reason for action. The social worker may, however, be able to work around the hospital's rules to meet the needs of the pt. Following the doctor's orders (answer B) is not correct as it is the hospital social worker's responsibility to attend to the. Needs of the pt. Following the social worker's own opinion (answer D) is not correct b/c the social worker should respect the self- determination of the pt.

A social work supervisor has chosen to use group supervision rather than individual supervision with his supervisees. All of the following statements about group supervision are true EXCEPT A. Group supervision involves the sharing of supervisory responsibility by the supervisor and the supervisees B. Group supervision is a time saver for supervisors C. Group supervision is a more effective approach to supervision than individual supervision D. Group supervision is often less threatening to supervisees

C- Group supervision is a more effective approach than individual supervision. The question asks the reader to identify the statement that is not accurate with regard to group supervision and only answer C is false. Group supervision is considered a helpful supplement to individual supervision. Individual supervision is the preferred single mode of supervision.

In a session with a woman suffering from depression the client recounts, in great detail, a dream she has in which she bought a gun and took her life. She tell the social worker specifically how she plans to carry out the dream. The social worker should A. Shear this information with the family B. Develop a no suicide contract C. Hospitalize the patient D. Ignore the dream

C- Hospitalize the patient. The pt has stated that she intends to commit suicide and she has a specific plan to achieve this goal. These conditions are associated with high risk for suicide. The woman needs to be hospitalized until she is not longer at risk. Sharing the information with the family (answer A) is not the best answer b/c, if it is appropriate, it will come after the client's safety has been secured through hospitalization. Developing a no- suicide contract (answer B) is not correct b/c developing a no- suicide contract is not an immediate enough plan and has questionable effectiveness. Ignoring the dream (answer D) is not correct b/c a responsible social worker should never ignore a dream as disturbing and concrete at this.

An individual who calls a crisis line begs in to tell the social worker that he is being followed and cannot determine who the person is who is following him. The caller also mentions that she was in the hospital about a week ago and is having a hard time leaving the house. The social worker should FIRST A. Obtain the name of the hospital B. Obtain the names of the caller's doctors C. Identify ways to obtain safety for the caller D. Obtain the caller's home address

C- Identify ways to obtain safety for the caller. The safety of the client is of paramount importance. Obtaining the name of the hospital (answer A), obtaining the names of the caller's doctors (answer B), and obtaining the caller's home address (answer D) are not the best answers b/c the crisis worker needs to obtain this other information after the safety have been addressed.

As individuals grow and develop they are influenced by their environment. Theorist Leon Chestang identified the systems of which everyone is a part. He asserted that the Sustaining system consists of A. The individual's family B. The "helping" profession C. Institutions of the larger society D. A physiological system that sustains life

C- Institutions of the larger society. This term is most associated with the culturally- related research of Leon Chestang. The Sustaining system is one in which the individual receives his education, is employed and is involved in the economic and political world. The individual's family (answer A) is not correct b/c this is associated with Chestang's "Nurturing system" classification. The Nurturing system is one in which a person is viewed and treated as a unique individual who is loved for who he is by his family, friends and neighborhood acquaintances in his immediate environment. The Sustaining system is not associated with the "helping professions" (answer B) per say, nor does the Sustaining system relate to a physiological system that sustains life (answer D).

A 16- year- old female is seeing a social worker Ina local runaway shelter for teenagers. She states that she is pregnant and has been using heroin quite frequently. The social worker's FIRST responsibility is to A. Ensure the safety of the unborn B. Provide information on contraceptive methods C. Make sure the mother's needs are met D. Locat her parents

C- Make sure the mother's needs are met. The pregnant teen is the social worker's client. Making sure her needs are met then must be the primary concern of the social worker. The social worker can help the unborn child (answer A) by helping the mother. Providing information on contraceptive methods (answer B) is not correct b/c the girl is already pregnant. Depending on the laws of the state, the social worker may then locate the client's parents (answer D).

A social worker is seeing a client who complains constantly of smells that aggravate him. Recently he has stated that he cannot sleep b/c of the smell of fungus in his home. His work is being affected b/c the fungus smell at his office makes it impossible for him to do his job. He has had his home inspected and no fungus has been found. His employer refuses to have the workplace fumigagted. He can no longer date his girlfriend b/c she smells of fungus. What is this client likely experiencing? A. Hyper Olfactory Senses Disorder B. Serious breathing problems C. Olfactory hallucinations D. Somatization Disorder

C- Olfactory hallucinations. The client is likely smelling something that does not really exist and it is interfering with his personal functioning, his work life, and his social life. Hyper olfactory Senses Disorder (answer A) is not correct as this is not a real condition. Serious breathing problems (answer B) is not correct b/c breathing problems indicate an inability to take air into the lungs. Somatization Disorder (answer D) is not correct b/c Somatization Disorder is characterized by pain in at least four different parts of the body or body functions; at least two, non-pain, gastrointestinal symptoms; at least one sexual or reproductive- oriented symptom; and at least one, non- pain, pseudo neurological symptom.

A social worker is meeting with a client for the first time. What types of questions are most likely to encourage the client to talk freely about the issues that have brought her into treatment? A. Leading questions B. Closed-ended questions C. Open- ended questions D. Stacked questions

C- Open- ended questions. Leading questions (answer A) is not correct b/c leading questions have an underlying goal of obtaining client agreement rather than the client's true perspective. Closed-ended questions (answer B) is not correct as closed- ended questions are used to obtain specific pieces of information and usually elicit a brief answer. Stacked questions (answer D) is not correct b/c stacked questions involve asking a series of questions in quick succession, which often leads to confusion. Both leading and stacked questions are questioning styles social workers should avoid.

A college student has made an appointment to speak with a counselor at her university's counseling center about troublesome symptoms she has experienced on a number of occasions over the last six months. The symptoms include chest pain, nausea, pounding heart, and dizziness. A recent medical exam failed to identify any physical problems. Which of the following DSM-5 diagnoses is this student MOST LIKELY suffering from? A. Generalized Anxiety Disorder B. Social Anxiety Disorder C. Panic Disorder D. Specific Phobia

C- Panic Disorder. Generalized Anxiety Disorder (answer A) is not correct b/c Generalized Anxiety Disorder is characterized by excessive anxiety about number of events or activities. Social Anxiety Disorder (answer B) is not correct b/c a Social Phobia is evoked by social or performance situations. Specific Phobia (answer D) is not correct b/c the diagnosis of Specific Phobia would be appropriate if these symptoms were evoked by a specific object or situation other than a social situation.

The DSM-5 defines which of the following categories of disorders as "an enduring pattern of inner experience and behavior that deviates markedly from the expectations of the individual's culture?" A. Anxiety Disorders B. Mood disorders C. Personality disorders D. Psychotic disorders

C- Personality disorders. Anxiety disorders (answer A) is not correct b/c anxiety disorders are characterized by fear, panic, and debilitating physical symptoms. Mood disorders (answer B) is not correct b/c mood disorders are characterized by depressive or manic symptoms. Psychotic disorders (answer D) is not correct as psychotic disorders are characterized by hallucinations, delusions, decrease in or disorganized speech, inappropriate or flat affect, disorganized behavior, and loss of ability to experience pleasure.

A social worker who lives in a large community is a recovering alcoholic and attends Alcoholics Anonymous meetings near his home. At one of the meeting a former client introduces himself as a new member of the AA group. What is the FIRST step that the social worker should take? A. Walk out of the meeting B. Acknowledge the client in front of the group C. Remain at the meeting but not acknowledge the client as a person he knows D. Talk to the group leader aft earth meeting and explain the dual relationshp

C- Remain at the meeting but not acknowledge the client as a person he knows. This is the best answer, as it protects the client's confidentiality and is the least disruptive action. Walking out of the meeting (answer A) is not correct b/c the client may feel rejected and the other group members may question the reason for the action. Acknowledging the client in front of the group (answer B) is not correct b/c this would violate the client's rights to privacy. Talking to the group leader after the meeting to explain the dual relationship (answer D) is not correct b/c this would also violate the client's right to confidentiality.

A new mother who has been seeing a social worker for postpartum depression tells the social worker that she has been thinking about drowning her baby and her sleep. In this situation, it is important for the social worker to A. Set up a no- harm contract B. Inform his or her supervisor of the situation C. Report the case to Child Protective Services D. Have the mother committed

C- Report the case to Child Protective Services. All states require mental health professionals to file a report with CPS if they have reason to believe that a child has been or is danger of being harmed. Setting up a no- harm contract (answer A) is not correct b/c the safety of the child is paramount and the mother is suffering from postpartum depression and not in control of her feelings and actions. Informing his or her supervisor of the situation (answer B) is not the best answer in this case, although informing the supervisor is a good step to take after fulfilling the legal requirement of reporting the situation to CPS. Having the mother committed (answer D) is not correct b/c after CPS evaluates the condition of the mother, it would be the CPS staff that can have her committed to the hospital if the action is warranted.

All of the following are components of crisis intervention EXCEPT A. Concentration on limited goals B. Problem- solving C. Resolution of unresolved issues from childhood D. Enhancement of the client's self- esteem

C- Resolution of unresolved issues from childhood.. Crisis intervention is a short- term form of treatment that does not focus on intrapsychic conflict from childhood but on increasing the individual's ability to cope through activities such as the development of limited goals, problem solving, and the enhancement of client self- esteem.

An 18- year- old Native American male from the Ute Tribe is seeking help from a social worker to deal with his alcohol use issues. His mother and grandmother make an appointment to talk to the social worker and express their concern about his seeing a social worker. They indicate that this is an embarrassment to the family and the tribe and they would like the social worker to stop seeing the man. What is the FIRST course of action should the social worker take? A. She should respect the family's culture and discontinue seeing the man B. She should tell the mother and grandmother that she will talk to the client and encourage him to get help on the reservation C. She should acknolwedge the family's concern and explain that the client is an adult and has the right to make his own decisions. D. She should contact the Ute Tribe Coucil for direction.

C- She should acknowledge the family's concerns but explain that the client is an adult and has the right to make his own decisions. Respecting the family's culture and discontinuing her contact with the the client (answer A) is not correct b/c the social worker must follow the NASW Code of Ethics and uphold the client's right to self- determination. While cultural sensitivity is an important factor in providing treatment, protecting the client's ethical rights should come first. Telling the mother and grandmother that she will talk to the client and encourage him to get help on the reservation (answer B) is no correct b/c the client has the right to seek help wherever he prefers. Contacting the Ute Tribe Council (answer D) is not correct b/c the social worker should follow the Code of Ethics first before taking any other action. Test Tip: This case scenario contains some distractors to confuse you. The Ute Tribal Council may have some influence over this man but you do not know that at this point and you should use your professional training with him. Follow what you know to be good social work practice. Stating that the client has alcohol use issues is also a distractors as that information may lead you to think in terms of stereotypes of Native American individuals. It would not matter what the nationality or presenting problem is in this scenario, adherence to the. Code of Ethics takes precedence in treatment decisions.

A social worker in a mental health agency has received a referral to provide services for a client who is court- ordered to receive social work treatment for criminal activity that led to his probation. The client does not want to participate in the treatment process and has told the social worker that he does not intend to keep his appointment. What should the social worker do? A. She should respect the client's right to self- determination and inform the probation officer that the client does not wish to participate in treatment. B. She should inform the client hat since he is court ordered, he has no option but to participate in the treatment C. She should provide the client with information about what services she can offer and let him know the extent of his right to refuse services D. She should encourage the client to try at least one session with her and then decide that he would like to do

C- She should provide the client with information about what services she can offer and let him know the extent of his right to refuse services. This is the procedure recommended by the NASW Code of Ethics. The social work would need to consult with the probation officer to determine what the client's rights or refusal are since he is court ordered, and provide that information to the client. Respecting the client's right to self- determination and information the probation officer that the client does not wish to participate in treatment (answer A) is not correct b/c the the client does have court- ordered restrictions regarding his participation in treatment. Information the client that since he is court- ordered he has no option but to participate in the treatment (answer B) is not correct b/c there may be conditions that allow him to have options. Encourage the client to try at least one session with her and then decide what he would like to do (answer D) is not correct b/c this is coercive and the decision should rest with the client.

A social worker in a detoxification facility discovers that a new patient told her former social worker that she brought prescription drugs in her lipstick case in the detox facility. Agency policy calls for the patient to start detoxification over from the beginning if drugs are found. However, the client states that she has not taken any drugs since being admitted. The social worker should A. Ignore the drugs and continue treatment B. Ask the patient to leave C. Start detoxification over agin since that is the policy D. Meet with other members of the facility to make a decision

C- Stat detoxification over since that is the policy. A policy of this type has been put in place to deal with this very kind of occurrence. Ignoring the drugs and continuing treatment (answer A) is not correct b/c ignoring the drugs would never be an appropriate response. Asking the pt to leave (answer B) is not correct b/c this would be an extreme course of action to take in this situation. Meeting with other members of the facility to make a decision (answer D) is not correct b/c this action would not be appropriate as the policy is already in place to deal with this kind of situation, so a further decision would not be necessary unless the policy were inappropriate.

A client displays dilation of pupils, dry mouth, and increased heart rate. From which division in the autonomic nervous system would these symptoms emerge? A. Central Nervous Division B. Adrenaline Division C. Sympathetic Division D. Parsympathetic Division

C- Sympathetic Division. The Autonomic Nervous System has two division: the Sympathetic and Parasympathetic. The symptoms described are associated with the Sympathetic Division. The Parasympathetic Division (answer D) functions is opposition to the Sympathetic Division (e.g inhibits heartbeat, contracts pupils).

There are many different types of groups that social workers lead. The types of group MOST associated with self- improvement and opportunities to expand self- awareness is A. The Therapeutic Grup B. The Socialization Group C. The Growth Group D. The Recreational Group

C- The Growth Group. The Therapeutic Group (answer. A) is not correct b/c a therapeutic group deals with and resolves personal problems. The Socialization Group (answer B) is not correct b/c a socialization group assists members in negotiating developmental stages and adopting to changes in roles of environment (e.g a group for new immigrants). The Recreational Group (answer D) is not correct b/c a recreational group is not lead by a social worker.

A social worker is working with a family that consists of a father, a mother, and three children, ages 10, 8, and 5. The father spends much of the session time criticizing other family members. The father;s behavior is consistent with which of the following styles of communication identified by Virginia Satir? A. The irrelevant B. The placater' C. The blamer D. The super- reasonable

C- The blamer. The blamer tends to accuse, criticize, and dominate interaction. The irrelevant (answer A) is not correct b/c the irrelevant is not a role of behavior identified by Satir. The placater (answer B) is not correct b/c the placater is characterized by agreement, apologizing, and efforts to please. The super- reasonable (answer D) is not correct b/c the super- reasonable maintains an outward appearance of clam, coolness, and emotional detachment.

Which of the following organizational tools indicates why the organization exists? A. Policy statements B. Organizational goals C. The mission statement D. Organizational procedures

C- The mission statement. Policy statements (answer A) is not correct b/c policy statements are general statements or understandings that serve as the basis for thinking and decision- making. Organizational goals (answer B) is not correct b/c organizational goals are general statement about what an organization wants to achieve. Organizational procedures (answer D) is not correct b/c organizational procedures are the specified ways an organization has of conducting business.

A social worker has been assigned to work with a client who has received a diagnosis of Antisocial Personality Disorder. To which treatment strategy would the client respond best? A. Medication B. Assertiveness training C. Treatment focused on emotions D. Psychoanalysis

C- Treatment focused on emotions. Since individuals diagnosed with Antisocial Personality Disorder have a history of disregard or violation of others' rights, treatment that helps the individual's to consider the feelings of others is most effective. Medication (answer A) is not correct b/c medication is not indicated for Antisocial Personality. Disorder. Assertiveness Training (answer B) is not correct b/c this would only reinforce the individual's problematic behavior. Psychoanalysis (answer D) is not correct b/c insight into one's early developmental experiences may not effectively address the destructive behavior that is directed toward others.

A mother who is trying to teach her son to pick up his toys should begin by using what schedule of reinforcement to ensure that this behavior becomes a habit for him A. Primary reinforcement B. Intermittent reinforcement C. Continuous reinforcement D. Shaping

C- continuous reinforcement or, in other words, reinforcement after every occurrence of the desired behavior. Continuous reinforcement is the most effective strategy in habit development. Primary reinforcement (answer A) is not correct as primary reinforcement refers to providing something that is required to sustain life (e.g water, food, sleep) but is often the procedure of giving an edible reward for appropriate behavior. Intermittent reinforcement (answer B) is not correct as intermittent reinforcement is reinforcement of only some occurrences of the target behavior. Shaping (answer D) is not correct as shaping is a technique of reinforcing successive approximations to the desired behavior.

During a rational emotive therapy (RET) intervention, the client describes a recent situation at work in which he responded to the suggestions of his boss with an angry report. The "C" is this situation is A. The boss's suggestion B. The boss C. The client's angry retort D. The conflict

C- the Client's angry retort. The "C" in RET refers to the emotional consequences (in this case, the client's angry retort) in response to the client's belief (B) and the incident that occurred at work (A). Albert Ellis, folding theorist for Rational Emotive Therapy, believes that an activating event does not cause a person to feel a certain way, but feelings are the consequences of the individual's beliefs about the event. Beliefs can be rational or irrational, and irrational beliefs lead to unnecessary painful emotions and maladaptive behavior. Emotional health results from the rational or logical processing of activating events.

In Pavlov's research, after the bell had been paired with the meat powder on repeated occasions, the dogs came to salivate in response to the bell in the absence of the meat powder. Salivation in response to the bell is A. An unconditional stimuli B. A conditioned stimuli C. An unconditioned response D. Conditioned response

D- A conditional response. The dogs learned through classical conditioning to salivate at the bell's tone. An unconditional stimuli (answer A) is not correct b/c unconditional stimuli refers to stimuli that innately evoke response to an organism. A conditional stimuli (answer B) is not correct b/c conditional stimuli do not innately evoke a response in an organism but that organism learns to respond to the stimuli b/c it has been paired in the past with an unconditional stimulus. An unconditional response (answer C) is not correct b/c an unconditional response is an innate response to a stimulus.

Psychostimulatns are used primarily to treat A. Depression B. Psychoses C. Anxiety D. ADHD

D- ADHD. Pscyostimulants have a paradoxical effect on many individuals with ADHD. Rather than creating wakefulness and increased alertness, they tend to improve their ability to focus and to exercise self- control. Depression (answer A) is not correct as symptoms of depression are treated with antidepressant medication. Psychosis (answer B) is not correct as antipsychotic (also called major tranquilizer so ready neuroleptics ) are used to treat psychoses. Anxiety (answer C) is not correct b/c anxiety is treated with anti- anxiety or anxiolytic medications.

While presenting a workshop at a conference on the topic of Personality Disorders, an attendee speak with a social worker at the break and describes herself as a Borderline Personality Disorder previously diagnosed at the psychiatrist's office. She asks the social worker to confirm the diagnosis. What the the BEST response for the social worker to take? A. Explain that she cannot do a diagnosis under the circumstances. If the attendee has concerns, she should check with her doctor. B. Meet wit the individual after the conference to discuss her concerns. C. Set an appointment to meet with the attendee D. Advise her to see another psychiatrist for a second opionion.

D- Advise her to see another psychiatrist for a second opinion. The woman is an attendee of a workshop, not the social worker's pt. If the woman feels a need for a confirmation of the diagnosis she was given, the social worker can best serve her by suggesting that she seek confirmation from another social worker or psychiatrist. Suggesting that she check back with her doctor (answer A) is not correct b/c this does not address her desire for a confirmation of the diagnosis. Meeting with this individual after the conference to discuss her concerns (answer B) is not correct b/c since she is not the social worker's client the social worker cannot discuss her concerns with her, nor set up an appointment with her (answer C).

An 18- year- old woman in treatment informs the social worker that she would like to have an abortion for her unexpected pregnancy. The FIRST step the social worker should take is to A. Refer the client elsewhere B. Refer the client to a doctor who performs abortions C. Provide alternatives to the client D. Aid the client in looking at all reasonable option open to her

D- Aid the client in looking at all reasonable options open to her. This answer is consistent with two important values of social worker: informed consent and self- determination. The social worker may ultimately refer the client elsewhere (answer A) or refer the client to a doctor who performs abortions, (answer B), but not until she assists the client in looking at her options. Providing alternatives to the client (answer C), in incorrect b/c it fails to involve the client in the process.

Which of the following individuals in an organization have advocacy responsibilities? A. Board members B. Client's C. Support staff D. All of the above

D- All of the above. Other members of organization's who have advocacy responsibilities include the director, front- line workers, and supersivors.

Sublimation can be seen in A. Cooking B. Woodworking C. Football D. All of the above

D- All of the above. Sublimation is a higher level ego defense mechanism involving the focusing of libidinal energy into creative and constructive activities. All of the listed activities: cooking (answer A), woodworking (answer B), and football (answer C) qualify.

The term used to refer to the impairment in language sometimes experienced by individuals with Alzheimer's is A. Apraxia B. Agnosia C. Abulia D. Aphasia

D- Aphasia. Apraxia (answer A) is not correct b/c Apraxia is an inability to perform major activities, although motor functions are intact. Agnosia (answer B) is not correct b/c Agnosia is an inability to recognize objects, although sensory functions are intact. Abulia (answer C) is not correct b/c Abulia ha reference to a loss of ability to make choices or a loss of will.

Token economies are most often used with A. Families B. Marital couples C. Children D. Groups

D- Children. A token economy is a behavioral technique, and behavior modification is very effective with children. Families (answer A), marital couples (answer B), and groups (answer D) are all incorrect b/c families, marital couples and groups are more successful with therapies that utilize relationships rather than behavioral techniques.

A client is seeing one social worker for individual therapy and another social worker for group therapy. The client is improving. The most likely reason is A. Combination of the two kinds of therapy B. Individual social work C. Group social work D. Communication b/w social workers

D- Communication b/w social workers. Communication of the two kinds of therapy (answer A) is not correct b/c in not all cases is a combination of individual and group therapy more helpful that none or the other;; it depends on the client's needs. Individual social worker (answer B) and group social worker (answer C) are not correct b/c, while the skills of the individual and group social worker may be important, it is the communication b/w the two social workers that can improve the treatment provided and facilitate more rapid improvement in the client.

A 60- year- old woman has recently been hospitalized with a delusional disorder. She is convinced that her neighbor is able to access her thoughts through the telephone lines. Which of the following approaches should the social worker NOT take in dealing with the individual's delusion.? A empathize with how vulnerable she must be feeling B. Acknowledge how difficult it must be to be hospitalized C. Acknowledge how real the delusions feels to the client D. Confront the client with the unreasonableness of the delusion

D- Confront the client with the unreasonableness of the delusion. When dealing with delusional client's a social worker should not try to reality test them. Instead, the social worker should deal with them in a calm, empathic manner. All of the other answers listed are appropriate approaches to take with the client.

A 28- year- old male and his wife have been in therapy for one year. In therapy law week the couple described some issues of concern to them. They discussed how the husband has not bathed in over a week. His speech is slower than normal and he seems to ramble. His emotional reactions to situations seem inappropriate under the circumstances. He has been ambivalent in he decision-making and looks out the window reportedly talking to his decreased mother during dinner while family members are trying to interact with him. The MOST LIKELY action that he social worker would take would be to A. Diagnose the husband with Schizophrenia B. Diagnose the husband with Major Depressive Mood Disorder C Diagnose the husband with Substance Induce Mood Disoder D. Continue to assess the situation

D- Continue to assess the situation. The question does not indicate that that the social worker has information about recent trauma, medical or substance use issues, etc. Much of the behavior appears to be recent and there is an obvious lack of needed information. Therefore, it is not appropriate to make a diagnosis at this time.

During a session with a 36- year- old female client, a social worker becomes angry in response to the woman's tone of voice. Very likely, the social worker is experiencing A. Empathy B. Reaction formation C. Transference D. Countertransference

D- Countertransference, which refers to a mental health provider responding to a client as if he or she is significant person from the provider's past. It is very possible that the tone of voice that elicited anger in the social worker reminded him or her of negative interactions with a parent during his or her childhood. Empathy (answer A) is not correct b/c empathy is the action of accurately reflecting the surface feelings of the client. Reaction formation (answer B) is not correct b/c reaction formation refers to a defense mechanism of adopting a behavior that is the antithesis of the instinctual urge (e.g acting as if one has deep sympathies for an oppressed group when the individual actually has significant prejudices against the group). Transference (answer. C) is not correct b/c transference refers to the client's reaction to the social worker as if the social worker were someone in the client's past.

Which diagnosis below is contained in the Schizophrenia Spectrum and other Psychotic Disorders chapter of the DSM-5? A. Hoarding B. Disruptive Mood Dysregulation. Disorder C. Body Dysprhoic Disorder D. Delusional Disorder

D- Delusional Disorder A- Hoarding Disorder is incorrect since it is contained in Obsessive- Compulsive and Related Disorders. B- Disruptive Mood Dysreuglation Disorder is part of the Depressive Disorders chapter. C- Body Dysphoric Disorder is also not correct. It is contained in the Obsessive- Compulsive and Related Disorders chapter.

A Youth Corrections social worker is holding weekly group therapy sessions with the residents of a group home. In establishing the ground rules for the group, all of the following are appropriate EXCEPT A. Respect other's comments B. Be on time to group C. Maintain confidentiality D. Dont' express negative emotions

D- Don't express negative emotions. The questions asks which of four possible ground rules for a group is not appropriate. It is critical that group members be encouraged to express their emotions, positive and negative, in appropriate ways.

A social worker has recently learned from the wife of a couple he is seeing that the wife believes that she is a lesbian. She states she does not want to share this with her husband. During the next session, the husband asks his wife if she likes women better than men. The wife answers "No". What is the FIRST response that the social worker should make in this situation? A. Encourage the wife to be honest B. Encourage the husband to ask his wife after the session C. Say nothing but plan on speaking with the I wife after the session D. Encourage the couple to discuss the issue

D- Encourage the couple to discuss the issue. The responsibility for establishing a relationship of honesty rests with the couple. The social worker can facilitate the process of developing such a relationship by encouraging dialogue about this and other important issues. Encouraging the wife to be honest (answer A) is not the best answer for this scenario, as the social worker would be indicating that he has information that the husband does not have. The social worker may, at another time, need to help the wife decide how she will handle this information with her husband. Encouraging the husband to ask the wife after the session (answer B) and saying nothing but planning on speaking with the wife after the session (answer C) are both incorrect answers b/c these would be interfering with the couple dealing with their own situation.

While working with a 16- year old girl in individual therapy, the mother (who is not the custodial parent) calls the social worker on the phone and wants to report on her daughter's depression. In this situation, the social worker should A. Inform the mother that she has no right to that information B. Share with the mother what is happening with the daughter C. Call the father to deal with the mother, as he has custodial rights D. Encourage the mother to talk with the daughter

D- Encourage the mother to talk with the daughter. This is the most sensitive way to deal with the issue. In addition, this response has the potential to strengthen the mother- daughter relationship. Although informing the mother that she has no right to that information (answer A) is technically correct, as the social worker cannot share information without permission form the custodial parent, the better answer is to encourage the mother to talk with the daughter. Sharing with the mother what is happening with the daughter (answer B) is not correct b/c the social worker cannot share this information with the noncustodial parent. Calling the father to deal with the mother, as he has custodial rights (answer C) is not correct b/c it would be better if the social worker suggested to the mother that she discuss the matter with the father rather than the social worker contacting the father and acting as a go- between.

A 9- year- old girl has been removed from her parent's home following a pattern of physical and verbal abuse. The family has not followed through on any of their contracts set up with the Division of Child and Family Services. The girl has remained in care for almost 16 months and is being considered for permanency planning outside the home. All of the following would be appropriate for the social worker to do EXCEPT A. Deal with non- reunification issues B. Play therapy C. Seek supportive services D. Family therapy for abuse issues

D- Family therapy for abuse issues. Family therapy is not correct b/c reunification is no longer part of the girl's permanency plan., and the question asks for an answer the would NOT be appropriate.

While working with a Native American family the social worker notices that the father is avoiding eye contact. The MOST LIKELY reason for this is A. His dishonest B. His low self- esteem C. His embarrassment D. His cultural differences

D- His cultural differences. The behavior of clients in counseling situations is often a function of their culture. His dishonesty (answer A) is not correct b/c there is no reason to think that the client is being dishonest. His low self- esteem (answer B) is not correct b/c while low self- esteem could be an issue, unless it is known, it is inappropriate to think that it is the reason for avoiding eye contact. His embarrassment (answer C) is incorrect b/c although the client's embarrassment could be a factor, the cultural differences should be the first consideration.

A 30- year- old woman is preoccupied with the fear that her body is filled with cancer even though doctors have assured her that she is in excellent physical health. This woman would most likely be diagnosed with A. Somatic Symptom Disorder B. Factitious Disorder C. Conversion Disorder D. Illness Anxiety Disoder

D- Illness Anxiety Disorder. Illness Anxiety Disorder involves preoccupation with fears that one has a serious medical condition. Somatic Symptoms Disorder (answer A) is not correct b/c the individual has actual somatic symptoms that are viewed as a sign of serious illness. Factitious Disorder (answer B) is not correct b/c Factitious Disorder is characterized by intentional deception that one has an injury or disease. Conversion Disorder (answer C) is not correct b/c Conversion Disorder is characterized by loss of functioning in voluntary motor or sensory functions not the result of a medical condition, a substance or a phenomenon sanctioned by a culture.

With Borderline Personality Disorder, it is most likely that the individual has a problem with which of the following defense mechanisms? A. Denial B. Sublimation C. Undoing D. Isolation of Affect

D- Isolation of Affect. Individuals with this diagnosis often defend by Compartmentalizing their emotions. Denial (answer A) is not correct b/c denial is more likely a defense mechanism associated with an anxiety disorder, as it is refusal to acknowledge an aspect of reality b/c to do so would result in overwhelming anxiety. Sublimation (answer B) is not correct, as sublimation is a health defense mechanism in which intolerable drives or desires are diverted in to activities which are acceptable. Undoing (answer C) is not correct b/c undoing is a defense mechanism more likely associated with Obsessive Compulsive Disorder, as it is a mechanism by which an individual engages in a repetitious ritual in an attempt to reviser an action previously taken.

A social worker is working with a client using a task- centered treatment model. This model is based on which of the following theories? A. Systems theory B. Learning theory C. Cognitive theory D. Learning and cognitive theory

D- Learning and cognitive theories. Systems theory (answer A) is not correct b/c systems theory is used in family and couples therapy. Learning theory (answer B) is not correct b/c learning theory alone is used in behavior modification. Cognitive theory (answer C) is not correct b/c cognitive theory is used in rational emotive therapy, self- management therapy, and cognitive therapy.

What is the treatment for children who are diagnosed with Autistic Disorder? A. Behavior modication B. Specialized diet that includes natural supplements C. Medication D. No single treatment that has been accepted as the best treatment for children with autismq

D- No single treatment has been accepted as the best treatment for children with autism. Every child is an individual and responds differently to treatment strategies. Behavior modification (answer A), specialized diet that includes natural supplements (answer B), and medication (answer C) are all appropriate options for treatment, but no one treatment has been accepted as the "best" treatment given the disorder's complexities and the lack of conclusive evidence regarding its cause.

A social worker has been hired as a consultant to a large organization that is experiencing administrative problems. Consultation in social work A. Is a one-way process in which an expert provides a non- expert with needed information B. Involves a series of stages that are moved through in a set sequence C. Rarely meets with resistance D. Often consists of the sharing of professional expertise relative to such things as personnel and budgets

D- Often consists of the sharing of professional expertise relative to such things as personnel and budgets. Stating that consultation is a one- way process in which an expert provides a non- expert with need information (answer A) is incorrect b/c consultation is a two- way process in which a less knowledge expert provides feedback to a more knowledgable expert which enables the latter to provide the former with the needed information. Stating that consultation involves a series of stages that are moved through in a set sequence (answer B) is not correct b/c the stages of the consultation process vary in terms of the sequence in which they are passed through form one consultation to another. Stating that consultation rarely meets with resistance (answer C) is incorrect b/c consultants are frequently viewed as intruders. An important part of the preliminary work of a consultant is often the identification of an organization's areas of resistance followed by efforts to reduce that resistance.

A social worker is employed in a community mental health agency. She is desirous of being an excellent social worker but wonders what factors result in the client being able to be successful. A large body of research has identified which of the following as the most important factor in therapy outcome? A. Theoretical orientation B. Intervention skills C. Client and worker homogeneity D. Quality of the therapeutic relationship

D- Quality of the therapeutic relationshp. Theoretical orientation (answer A) is not correct b/c experienced clinicians with different theoretical orientations seem to have comparable outcomes. Intervention skills (answer B) is not correct b/c skill level, though a factor in therapy outcome, has not been shown to be as important a factor as the quality of the therapeutic relationship. Client and worker homogeneity (answer C) is not correct b/c the important of homogeneity in client and worker characteristics is unclear.

A social worker has feelings of disgust and hatred toward pedophiles. She has chosen to work with people who have a record for child molestation and are court- ordered into therapy. Which of the following defense mechanisms might the social worker be utilizing? A. Denial B. Displacement C. Projection D. Reaction Formation

D- Reaction Formation. Reaction Formation means adopting a behavior that his the antithesis of the instinctual urge. The social worker has extreme negative feelings about pedophiles so she is choosing to work with them (I.e doing the opposite of what she actually believes). Denial (answer A) is not correct b/c Denial is a refusal to acknowledge an aspect of reality b/c to do so would result in overwhelming anxiety. Displacement (answer B) is not correct b/c Displacement is a shifting of negative feelings one has about a person or situation onto a different person or situation. Projection (answer C) is not correct b/c Projection means that one's own negative characteristics are denied and instead seen as being characteristics of someone else.

A grandmother brings her 6- year- old grandchild in for a consultation. The child exhibits a lack of awareness of the presence of others in the room, an inability to communicate, and a repetitive habit of waiving his hands in the air. This child has a disorder that is most often associated with what chemical in the body? A. Acetylcholine B. Catecholamines C. Endorphin D. Serotonin

D- Serotonin. This child has symptoms that are consistent with the diagnosis of Autistic Spectrum Disorder. This research currently links increased Serotonin levels, not the other answer options listed, with Autism Spectrum Disorder.

A social worker in a private mental health clinic has been asked to turn all of his records for a particular client over to the director's administrative assistance. There appears to be a family relationship between the director and the husband of the client. The social worker needs to A. Contact the client B. Refuse, as this is an unethical situation C. Turn the records over D. Speak with the director regarding the reasons for the request

D- Speak with the director regarding the reasons for the request. It is always important to clarify issues, in this case asking thee director about the reasons for the request, before taking action. The social worker may then need to contact the client (answer A) or turn the records over (answer C), depending on the reasons for the request or the agency policy. If the social worker determines that this is an unethical situation, he may need to take other action to refuse the request (answer B).

A social worker is interested in conducting a study of drug use among adults of different ethnic backgrounds. Which of the following sampling strategies is most apt to yield a sample that reflects the ethnic composition of the area? A. Simple random sampling B. Convenience sampling C. Snowball sampling D. Stratified sampling

D- Stratified sampling. This form of sampling is used to ensure that there are sufficient cases representing different values of a variable (in this case ethnic groups). Simple random selection of a predetermined number of cases from a sampling frame, where each case in the sampling frame has an equal probability of being selected for inclusion in the sample. Convenience sampling (answer B) is not correct b/c convenience sampling refers to cases that are selected for inclusion in a study sample b/c they can be readily accessed by the researcher. Snowball sampling (answer C) is not correct b/c snowball sampling is a sampling strategy that involves compiling the sample as the study progresses.

An older couple, with no children, comes to talk with a social worker in a local family service agency and works through many different issues during their treatment. They then begin to refer different other older couples who also have no children to the social worker. The social worker enjoys doing couples work but does not want to fill her schedule with only couples. The social work would also like to work with families. Since there are such a large number of couples being referred to the social worker, it is very difficult for her to continue working with families. Int his instance the social worker needs to A. Refer couples to other social workers B. Examine her issues regarding couples C. Work with whoever contacts her D. Talk with her supervisor about how to handle the situation

D- Talk with her supervisor about how to handle the sitatuion. When confronted with an issue of this type in an agency setting, it is typically a good idea to confer with one's supervisor. Following consultation with the supervisor, the social worker may decide to refer couples to other social workers (answer A). Examining her issues regarding couples (answer B) is not the best answer b/c there is not information in this question to indicate that the social worker has an issue with working with couples; she may simply want to expand her experience by working with different kinds of clients, including families. Working with whoever contacts her (answer C) is not the best answer b/c the social worker can decide with whom she wants to work, depending on the policies of the agency. Test tip: In this question the information that this was an "older" couple was put in as a distractors. Age has nothing to do with the question. IN the licensing exam, distractors such as this may be included in questions to elicit an emotional response from the test taker or to confuse the real intent of the question.

A social worker seeing a couple in marital therapy discovered, from a confidential phone call the husband makes to the social worker, that the husband is having an affair. The social worker should A. Tell the wife B. Tell the husband to tell the wife the truth C. Stop therapy D. Tell the husband that marital therapy requires the two committed individuals

D- Tell the husband that marital therapy requires two committed individuals. It is appropriate for the social worker to inform the husband of the essential ingredients of successful marital therapy. It is not the social worker's responsibility to tell the wife of the affair (answer A), nor is it appropriate to tell the husband what he should do in this situation (answer B). To stop therapy (answer C) is not correct b/c a social worker should not stop marital therapy just b/c of this new information.

Which of the following best differentiates between Autistic Disorder and Childhood Disintegrative Disorder? A. Impairment in expressive language B. Impairment in social skills C. Impairment in interpersonal relationships D. The loss of previously acquired skills

D- The loss of previously acquired skills. Impairment ini expressive language, social skills and interpersonal relationships are characteristics of both disorders. Only Childhood Disintegrative Disorder is characterized by normal development initially with later loss of previously acquired skills.

A client has been seeing a social worker in private practice and is benefiting from the treatment. The client recently lost her job and, therefore, her medical insurance. She wants to continue treatment but cannot afford to do so. In an effort to help her, the social worker has discussed options. Which of the following is the BEST option? A. In lieu of payment, the social worker will continue to see the client if she will. Help him edit a book he is writing. B. In lieu of payment, the social worker will continue to see the client if she promises that she will not tell anyone. C. The social worker will continue to see the client at no cost until the client obtain medical insurance. D. The social worker will refer the client to a mental health agency where she can receive low- cost social work services.

D- The social worker will refer the client to a mental health agency where sh can receive low- cost social work services. Continuing to see the social worker if the client helps him edit a book he is writing (answer A) is not correct b/c this would be an ethical violation. Continuing to see the social worker if the client promises that she will not tell anyone (answer B) is not correct b/c this is a threat and an exploitation of a client. Continuing to see the social worker at no cost until the client obtained medical insurance (answer C) is not the best answer b/c social worker have the right to be paid for their services and it may be a long time before the client's financial situation changes.

A school social worker in a public high school has been asked by her principal to set up a group for students who have been arrested for using drugs. He asks her to include some of the student leaders who have not used drugs, as positive rule models for the other students. Which statement best describes this type of group? A. This is a therapy group. B. This is an educational group. C. This is a support group. D. This group is not a good idea.

D- This group is not a good idea. There are too many concerns to justify this as an appropriate group for a public high school. Confidentiality could easily be violated, as group members would be expected to disclose their behavior, and it would be difficult to keep other students from knowing about the group. Research has shown that when drug users get together in an uncontrolled environment they can share drug information with one another, locate drug dealers, and sell to one another. They may also like to repeat their experiences in an affront to appear tough and sophisticated. Including non-drug users in a group also creates a risk of providing unnecessary information about drug use and seeking, glamorizing the behavior, and disappointing the student leaders when they find out information about the other students. The social worker should explain to her principal the risks involved and the violation of the NASW Code of Ethics. If there are students in the school who have been arrested for drug use, they should be in treatment with their Youth Corrections social workers.. Therapy groups (answer A) provide members with remediation and/ or rehabilition. Educational groups (answer B) help members learn specific information and skills that will be personally beneficial. Support groups (answer C) provide assistance in dealing effectively with common problems or circumstances. All of these groups would be appropriate for drug uses who have been arrested if they contain the right composition and are held in a therapeutic setting. A public high school is not the place for this to happen.

As a family social worker, a social worker in a mental health agency is aware of the issues regarding involuntary client's. A couple comes to the social worker to talk about their problems but admits that their primary reason for coming is their family's insistence to do so. The BEST action to take is A. To reschedule an appointment with the rest of the family B. Finish the session and explain that they will not be able to met with the social worker again unless it is entirely their own decision C. To cancel the session D. To encourage the couple to determine what they would like to achieve in therapy

D- To encourage the couple to determine what they would like to achieve in therapy. The couple has opted to seek counseling services. It is the social worker's responsibility to assist the couple in determining their goals for therapy. Scheduling an appointment with the rest of the family (answer A) is not correct b/c at this point, the social worker does not need to schedule an appointment with the rest of the family. Finishing the session and explaining that the couple will not be able to meet with the social worker again unless it is entirely their own decision (answer B) is not correct b/c it would not be helpful to the couple to discontinue service unless they have expressed that is their desire. Canceling the session (answer C) is not correct b/c it would be neither appropriate nor helpful to cancel the session.

A marriage and treatment clinic is offering couples group therapy sessions, to the conducted by a social worker. The focus of the group is to discuss the marital problems of the couple's with the hope that the couple's can provide support and suggestions to one another. While the social worker is conducting the first session with the group, a member of the group expresses a concern over the relatively low number of male participants in the group. The social worker's response should be A. To discuss men's reluctance to seek help B. To allow the men in the group to respond C. To agree with the group member D. To encourage the group to discuss this issue

D- To encourage the group to discuss this issue. A primary purpose of a facilitator in the early stages of a group is to help members create a cohesive group that can work together to achieve common goals. Toward this end, group facilitators need to encourage the members of the group to dialogue with each other about issues raised by individual members. Discussing men's reluctance to seek help (answer A) is not correct b/c is not relevant in this situation. Allowing the men in the group to respond (answer B) is not the best answer as the whole group needs to discuss the issue, and the men in the group should be encouraged to respond along with the other group members. Agreeing with the group member (answer C) is not correct b/c it is not appropriate for the group facilitator to agree with the group member er who expresses concern over the relatively low number of male participants; and is up to the other group members.

In the 1976 landmark case of Tarasoff v. Regents of the University of California, the decision by the court was an effort to ensure that A. Confidentially is maintained B. Suicidal client's be detained C. Substance abuse is not a valid argument for murder D. The individual is protected before confidentiality is protected

D- the individual is protected before confidentiality is protected. The other answer options are not correct as they are not relevant to the Tarasoff case, which was about harm to another person.

A social worker in a mental health agency decides to form a therapy group for individuals who are having marital difficulties. The social worker has asked colleagues to refer individuals to the group. In forming this group, the social worker will need all of the following information about the individual's EXCEPT A. The nature of their problem B. Their age C. Their stage in life D. The number of children they have

D- the number of children they have. This is not an important factor in marital counseling. The nature of the difficulties a couple is experiencing (answer A), their ages (answer B), and their stage in life (answer C) are all- important factors in working with married couples.


Related study sets

Chapters 1-4: End of Chapter Review

View Set

Chapter 13: Capacity to Contract

View Set

A&P2 Blood Vessels, A/P-Blood Vessels/Anatomy of Blood Vessels, A and P blood vessels, A&P Heart & Blood Vessels, Mastering A&P blood vessels, A & P Lab blood and vessels, A&P blood vessels and immune, A&P Blood vessels, Blood Vessels A&P, A&P Blood...

View Set

Ch. 26 - Male Genitalia and Rectum

View Set

Finding Slope/Y-Int. from Standard and Slope-Intercept Form

View Set

Chapter 8 - Supporting Your IdeasAssignment

View Set